Urology
Urology
gathered by dr. elbarky, for free, not intended for profit by anybody elsewhere
A 67 year old man presents with recurrent episodes of haematuria, typically at the
end of the urinary stream, he has been suffering from occasional fevers and has
noticed pus on the urethral meatus on occasion. On examination the prostate has
no discernable masses but is tender. What is the most likely cause?
Prostatitis
Cystitis
BPH
Prostate cancer
Nephritis
This is most likely prostatitis and the bleeding at the end of micturition suggests a
distal problem. Treatment is usually with prolonged courses of antibiotics.
Next question
BPH is increasingly common with advancing age and is present in 50% over the
age of 60 and nearly 90% of men by the age of 90. It occurs as a result of
hyperplasia of the periurethral glands in the transitional zone of the prostate.
Androgens play a role in the development and progression of BPH. Testosterone
diffuses into prostatic and stromal cells. Within epithelial cells it binds to the
androgen receptor. In prostatic stromal cells, a small proportion binds directly to
the androgen receptor, the majority binds to the 5 alpha reductase type II receptor
on the nuclear membrane. This is converted to dihyroxytestosterone and then
binds to the androgen receptor. Dihydroxytestosterone has even greater affinity for
the androgen receptor than testosterone does. The end result is stimulation of
these cells and proliferation.
This proliferative activity results in varying degrees of obstruction and results in
lower urinary tract obstructive symptoms. The clinical diagnosis of BPH thus
comprise a degree of lower urinary tract symptoms, palpable prostatic
enlargement and evidence of impaired voiding on urodynamic assessment.
gathered by dr. elbarky, for free, not intended for profit by anybody elsewhere
Presentation
Some patients have minimal symptoms, yet, on examination, have a palpable
bladder and obstructive post renal failure. LUTS can be divided into two main
groups; obstructive symptoms with voiding that include hesitancy, poor stream,
straining, prolonged micturition and dribbling; others develop irritation symptoms
which include pain during bladder filling, frequency, urgency and nocturia. Some
present with retention and haematuria.
Diagnosis
Abdominal and rectal examination
Symptoms scoring
Urodynamic studies ( a post void volume of >100ml is significant)
Management
Conservative
Alpha adrenergic antagonists. These block the action of noradrenaline on prostatic
smooth muscle causing relaxation and improved bladder emptying.
5 alpha reductase inhibitors. Finasteride blocks the enzyme 5 alpha reductase
which inhibits the conversion of testosterone to DHT. This in turn reduces
intracellular activity and decreases prostatic volume
Surgery- TURP is the gold standard. Occasionally, an open retropubic
prostatectomy may be considered for a large gland.
Next question
Save my notes
Question stats
A 68.9%
B 12.3%
C 6.2%
Question 2 of 63
gathered by dr. elbarky, for free, not intended for profit by anybody elsewhere
A 15 year old boy develops sudden onset of pain in the left hemiscrotum. He has
no other urinary symptoms. On examination, the superior pole of the testis is
tender and the cremasteric reflex is particularly marked. What is the most likely
underlying diagnosis?
Epididymo-orchitis
Viral orchitis
Testicular malignancy
The cremasteric reflex is usually preserved when the torsion affects the
appendage only.
Next question
Testicular disorders
Testicular cancer
Testicular cancer is the most common malignancy in men aged 20-30 years.
Around 95% of cases of testicular cancer are germ-cell tumours. Germ cell
tumours may essentially be divided into:
Tumour
Tumour type Key features markers Pathology
Tumour
Tumour type Key features markers Pathology
(https://d2zgo9qer4wjf4.cloudfront.net/images_eMRCS/swb090b.jpg)
Image sourced from Wikipedia
(https://d2zgo9qer4wjf4.cloudfront.net
(http://en.wikipedia.org
/images_eMRCS/swb090b.jpg)
/wiki/Seminoma)
Features
A painless lump is the most common presenting symptom
Pain may also be present in a minority of men
Other possible features include hydrocele, gynaecomastia
Diagnosis
Ultrasound is first-line
CT scanning of the chest/ abdomen and pelvis is used for staging
Tumour markers (see above) should be measured
gathered by dr. elbarky, for free, not intended for profit by anybody elsewhere
Management
Orchidectomy (Inguinal approach)
Chemotherapy and radiotherapy may be given depending on staging
Abdominal lesions >1cm following chemotherapy may require
retroperitoneal lymph node dissection.
Benign disease
Epididymo-orchitis
Acute epididymitis is an acute inflammation of the epididymis, often involving the
testis and usually caused by bacterial infection.
Infection spreads from the urethra or bladder. In men <35 years, gonorrhoea
or chlamydia are the usual infections.
Amiodarone is a recognised non infective cause of epididymitis, which
resolves on stopping the drug.
Tenderness is usually confined to the epididymis, which may facilitate
differentiating it from torsion where pain usually affects the entire testis.
Testicular torsion
Twist of the spermatic cord resulting in testicular ischaemia and necrosis.
Most common in males aged between 10 and 30 (peak incidence 13-15
years)
Pain is usually severe and of sudden onset.
Cremasteric reflex is lost and elevation of the testis does not ease the pain.
Treatment is with surgical exploration. If a torted testis is identified then
both testis should be fixed as the condition of bell clapper testis is often
bilateral.
Hydrocele
Presents as a mass that transilluminates, usually possible to 'get above' it
on examination.
In younger men it should be investigated with USS to exclude tumour.
In children it may occur as a result of a patent processus vaginalis.
Treatment in adults is with a Lords or Jabouley procedure.
Treatment in children is with trans inguinal ligation of PPV.
Next question
gathered by dr. elbarky, for free, not intended for profit by anybody elsewhere
Save my notes
Question stats
A 20.7%
B 24.2%
C 43.3%
D 6.7%
E 5%
Search eMRCS
Search term Go
External links
+ Suggest a link
Dashboard
3
Question 3 of 63
gathered by dr. elbarky, for free, not intended for profit by anybody elsewhere
A 22 year old man is involved in a road traffic accident. He is found to have a pelvic
fracture. While on the ward the nursing staff report that he is complaining of lower
abdominal pain. On examination, you find a distended tender bladder. What is the
most likely diagnosis?
Bladder rupture
Ureter injury
Urethral injury
Clot retention
Prostate rupture
Pelvic fractures may cause laceration of the urethra. Urinary retention, blood at the
urethral meatus and a high riding prostate on digital rectal examination are the
typical features.
Next question
Types of injury
Urethral injury Mainly in males
Blood at the meatus (50% cases)
There are 2 types:
gathered by dr. elbarky, for free, not intended for profit by anybody elsewhere
i.Bulbar rupture
- most common
- straddle type injury e.g. bicycles
- triad signs: urinary retention, perineal haematoma,
blood at the meatus
ii. Membranous rupture
- can be extra or intraperitoneal
- commonly due to pelvic fracture
- Penile or perineal oedema/ hematoma
- PR: prostate displaced upwards (beware co-existing
retroperitoneal haematomas as they may make
examination difficult)
Next question
Save my notes
Question stats
gathered by dr. elbarky, for free, not intended for profit by anybody elsewhere
A 16.8%
B 7.2%
C 51.9%
D 16.7%
E 7.3%
Search eMRCS
Search term Go
External links
+ Suggest a link
Dashboard
9
Question 4 of 63
gathered by dr. elbarky, for free, not intended for profit by anybody elsewhere
A 32 year old male presents with a swollen, painful right scrotum after being
kicked in the groin area. There is a painful swelling of the right scrotum and the
underlying testis cannot be easily palpated. What is the best course of action?
Inguinal orchidectomy
Scrotal orchidectomy
Scrotal exploration
Testicular USS
Testicular aspiration
Acute haematocele: tense, tender and non transilluminating mass. The testis will
need surgical exploration to evacuate the blood and repair any damage.
Next question
Scrotal swelling
Differential diagnosis
Management
Testicular malignancy is always treated with orchidectomy via an inguinal
approach. This allows high ligation of the testicular vessels and avoids
exposure of another lymphatic field to the tumour.
Torsion is commonest in young teenagers and the history in older children
can be difficult to elicit. Intermittent torsion is a recognised problem. The
treatment is prompt surgical exploration and testicular fixation. This can be
achieved using sutures or by placement of the testis in a Dartos pouch.
Varicoceles are usually managed conservatively. If there are concerns about
testicular function of infertility then surgery or radiological management can
be considered.
Epididymal cysts can be excised using a scrotal approach
Hydroceles are managed differently in children where the underlying
pathology is a patent processus vaginalis and therefore an inguinal
approach is used in children so that the processus can be ligated. In adults
gathered by dr. elbarky, for free, not intended for profit by anybody elsewhere
Next question
Save my notes
Question stats
A 6.9%
B 6.4%
C 43.6%
D 32.4%
E 10.7%
Search eMRCS
Search term Go
External links
+ Suggest a link
Question 5 of 63
gathered by dr. elbarky, for free, not intended for profit by anybody elsewhere
A 34 year old woman from Chad presents with continuous dribbling incontinence
after having her 2nd child. Apart from prolonged labour the woman denies any
complications related to her pregnancies. She is normally fit and well. What is the
most likely diagnosis?
Vesicovaginal fistula
Colovesical fistula
Pudendal neuropathy
Next question
Urinary incontinence
Involuntary passage of urine. Most cases are female (80%). It has a prevalence of
11% in those aged greater than 65 years. The commonest variants include:
Stress urinary incontinence (50%)
Urge incontinence (15%)
Mixed (35%)
Males
Males may also suffer from incontinence although it is a much rarer condition in
men. A number of anatomical factors contribute to this. Males have 2 powerful
sphincters; one at the bladder neck and the other in the urethra. Damage to the
bladder neck mechanism is a factor in causing retrograde ejaculation following
prostatectomy. The short segment of urethra passing through the urogenital
diaphragm consists of striated muscle fibres (the external urethral sphincter) and
gathered by dr. elbarky, for free, not intended for profit by anybody elsewhere
Females
The sphincter complex at the level of bladder neck is poorly developed in females.
As a result the external sphincter complex is functionally more important, its
composition being similar to that of males. Innervation is via the pudendal nerve
and the neuropathy that may accompany obstetric events may compromise this
and lead to stress urinary incontinence.
Innervation
Somatic innervation to the bladder is via the pudendal, hypogastric and pelvic
nerves. Autonomic nerves travel in these nerve fibres too. Bladder filling leads to
detrusor relaxation (sympathetic) coupled with sphincter contraction. The
parasympathetic system causes detrusor contraction and sphincter relaxation.
Overall control of micturition is centrally mediated via centres in the Pons.
Urethral mobility:
Pressure not transmitted appropriately to the urethra resulting in involuntary
passage of urine during episodes of raised intra-abdominal pressure.
Sphincter dysfunction:
Sphincter fails to adapt to compress urethra resulting in involuntary passage of
urine. When the sphincter completely fails there is often to continuous passage of
urine.
Urge incontinence
In these patients there is sense of urgency followed by incontinence. The detrusor
muscle in these patients is unstable and urodynamic investigation will
demonstrate overactivity of the detrusor muscle at inappropriate times (e.g.
Bladder filling). Urgency may be seen in patients with overt neurological disorders
and those without. The pathophysiology is not well understood but poor central
and peripheral co-ordination of the events surrounding bladder filling are the main
processes.
Assessment
Careful history and examination including vaginal examination for cystocele.
Bladder diary for at least 3 days
Consider flow cystometry if unclear symptomatology or surgery considered and
diagnosis is unclear.
Exclusion of other organic disease (e.g. Stones, UTI, Cancer)
gathered by dr. elbarky, for free, not intended for profit by anybody elsewhere
Management
Conservative measures should be tried first; Stress urinary incontinence or mixed
symptoms should undergo 3 months of pelvic floor exercise. Over active bladder
should have 6 weeks of bladder retraining.
Drug therapy for women with overactive bladder should be offered oxybutynin (or
solifenacin if elderly) if conservative measures fail.
In women with detrusor instability who fail non operative therapy a trial of sacral
neuromodulation may be considered, with conversion to permanent implant if
good response. Augmentation cystoplasty is an alternative but will involve long
term intermittent self catheterisation.
In women with stress urinary incontinence a urethral sling type procedure may be
undertaken. Where cystocele is present in association with incontinence it should
be repaired particularly if it lies at the introitus.
NICE guidelines
Initial assessment urinary incontinence should be classified as stress/urge
/mixed.
At least 3/7 bladder diary if unable to classify easily.
Start conservative treatment before urodynamic studies if a diagnosis is
obvious from the history
Urodynamic studies if plans for surgery.
Stress incontinence: Pelvic floor exercises 3/12, if fails consider surgery.
Urge incontinence: Bladder training >6/52, if fails for oxybutynin
(antimuscarinic drugs) then sacral nerve stimulation.
Pelvic floor exercises offered to all women in their 1st pregnancy.
Next question
Save my notes
Question stats
A 40.5%
Question 6 of 63
gathered by dr. elbarky, for free, not intended for profit by anybody elsewhere
A 23 year old man presents with severe loin to groin pain. Imaging demonstrates a
2mm left sided calculus in the distal ureter, renal function is normal. What is the
most appropriate course of action?
1mm 85%
2-4mm 75%
5-7mm 60%
Distally sited stones are more likely to pass spontaneously than proximally sited
ones.
Next question
Management
Most renal stones measuring less than 5mm in maximum diameter will typically
pass within 4 weeks of symptom onset(1)(2). More urgent treatment is indicated in
the presence of ureteric obstruction, renal developmental abnormalities such as
horseshoe kidney and previous renal transplant. Ureteric obstruction together with
infection is a surgical emergency and the system must be decompressed. Options
include nephrostomy tube placement or ureteric stent placement via cystoscopy.
In the non-emergency setting the preferred options for treatment of stone disease
include extracorporeal shock wave lithotripsy (ESWL), percutaneous
nephrolithotomy (PCNL) and ureteroscopy (URS). These minimally invasive options
are the most popular first line treatments. Open surgery remains an option in a few
selected cases.
Ureteroscopy
A ureteroscope is passed retrograde via the urethra and bladder, into the ureter and
renal pelvis. Laser or pneumatic fragmentation (lithoclasty) is performed using the
ureteroscope and stone fragments are extracted(3). In some cases a stent is left in
situ after the procedure.
Percutaneous nephrolithotomy
In this procedure percutaneous access is gained to the renal collecting system.
Once access is achieved, endoscopic intra-corporeal lithotripsy, lithoclasty or laser
stone fragmentation is performed and stone fragments removed.
Therapeutic selection
Renal stones
Ureteric stones
5-10mm ESWL
10-20mm Ureteroscopy
Reference
1. Bultitude M, Rees J. Management of renal colic. BMJ. 2012;345(7872):18.
2. Shah TT, Gao C, Peters M, Manning T, Cashman S, Nambiar A, et al. Factors
associated with spontaneous stone passage in a contemporary cohort of patients
presenting with acute ureteric colic: results from the Multi-centre cohort study
evaluating the role of Inflammatory Markers In patients presenting with acute
ureteric . BJU Int. 2019 Apr.
3. BAUS. Ureteroscopy (Telescopic surgery for stone removal) (Internet). BAUS
patient information leaflet. 2017 (cited 2019 Jun 27). p. 16. Available from:
https://www.baus.org.uk/userfiles/pages/files/Patients/Leaflets/Ureteroscopy for
stone.pdf
4. BAUS. Percutaneous nephrolithotomy (keyhole surgery for kidney stones). BAUS
patient information leaflet. 2017. p. 16. Available from: https://www.baus.org.uk
/userfiles/pages/files/Patients/Leaflets/PCNL.pdf
5. NICE. Renal and ureteric stones: assessment and management. NICE
Guide[NG118]. 2019;(January):112.
Next question
Save my notes
Question stats
gathered by dr. elbarky, for free, not intended for profit by anybody elsewhere
A 12.7%
B 5.7%
C 7.5%
D 17.2%
E 56.9%
Search eMRCS
Search term Go
External links
+ Suggest a link
Dashboard
9
Question 7 of 63
gathered by dr. elbarky, for free, not intended for profit by anybody elsewhere
Haematoma
Sperm granuloma
Varicocele
Hydrocele
Epididymal cyst
Sperm granulomas are a common sequelae of vasectomy and are smooth round
lumps adherant to the vas. They may be safely left alone.
Next question
Vasectomy
Procedure
Small bilateral incisions and formal dissection of the vas is the standard
technique. A technique involving the use of haemostats for skin puncture is used
in the 'no scalpel'
technique. It is not necessary to routinely send the vas for histology.
Controversies
Be wary of performing the procedure in childless, single men under age 30.
gathered by dr. elbarky, for free, not intended for profit by anybody elsewhere
Risks
Following the procedure between 12 and 52% of men reported chronic scrotal
pain. Of which 5.2% sought help. Haematomas and sperm granulomas may also
occur.
Follow up
Viable sperm may persist following surgery. Clearance should not be granted until
a negative sperm sample is available. This is usually taken after 12-16 weeks post
procedure (and preferably after 24 ejaculates). Recanalisation may occur in 0.04%
of cases.In a small minority of men, non-motile sperm persist after vasectomy. In
such cases, 'special clearance' to stop contraception may be given when less than
10,000 non-motile sperm/mL are found in a fresh specimen examined at least 7
months after vasectomy.
The risks of STI's are unchanged.
Next question
Save my notes
Question stats
A 10.6%
B 57.4%
C 7.9%
D 6.7%
E 17.3%
Search eMRCS
Question 8 of 63
gathered by dr. elbarky, for free, not intended for profit by anybody elsewhere
A 32 year old male presents with a swollen right scrotum which has developed
over 3 weeks after being kicked in the groin area. There is a non tense swelling of
the right scrotum and the underlying testis cannot be easily palpated. A dipstick is
positive for nitrates only. What is the most likely diagnosis?
Epididymo-orchitis
Testicular torsion
Hydrocele
Haematocele
Haematoceles develop over minutes/ hours and occur after the event
Next question
Scrotal swelling
Differential diagnosis
Inguinal hernia If inguinoscrotal swelling; cannot 'get above it' on
examination
Cough impulse may be present
gathered by dr. elbarky, for free, not intended for profit by anybody elsewhere
May be reducible
Management
Testicular malignancy is always treated with orchidectomy via an inguinal
approach. This allows high ligation of the testicular vessels and avoids
exposure of another lymphatic field to the tumour.
Torsion is commonest in young teenagers and the history in older children
gathered by dr. elbarky, for free, not intended for profit by anybody elsewhere
Next question
Save my notes
Question stats
A 9.1%
B 6.7%
C 7.3%
D 32.3%
E 44.7%
Search eMRCS
Search term Go
Question 9 of 63
gathered by dr. elbarky, for free, not intended for profit by anybody elsewhere
Which of the following renal stone types is most radiodense on a plain x-ray?
Calcium phosphate
Calcium oxalate
Uric acid
Struvite
Cystine
Calcium phosphate stones are the most radiodense stones, calcium oxalate
stones slightly less so. Uric acid stones are radiolucent (unless they have calcium
contained within them).
Next question
Renal stones
Type of Percentage of
stones Features all calculi
Type of Percentage of
stones Features all calculi
oxalate causes)
Hyperoxaluria may also increase risk
Hypocitraturia increases risk because citrate
forms complexes with calcium making it more
soluble
Stones are radio-opaque (though less than
calcium phosphate stones)
Hyperuricosuria may cause uric acid stones to
which calcium oxalate binds
Next question
Save my notes
Question stats
A 40.6%
B 33.3%
C 8.2%
D 9.8%
E 8.1%
Search eMRCS
Question 10 of 63
gathered by dr. elbarky, for free, not intended for profit by anybody elsewhere
An 83 year old man with a long standing staghorn calculus presents with recurrent
haematuria and investigation shows a mass of the left renal pelvis. Of the lesions
listed below, the diagnosis is:
Adenocarcinoma
Sarcoma
Transitional metaplasia
Next question
Renal tumours
Investigation
gathered by dr. elbarky, for free, not intended for profit by anybody elsewhere
Many cases will present as haematuria and be discovered during diagnostic work
up. Benign renal tumours are rare, so renal masses should be investigated with
multislice CT scanning. Some units will add an arterial and venous phase to the
scan to demonstrate vascularity and evidence of caval ingrowth.
CT scanning of the chest and abdomen to detect distant disease should also be
undertaken.
Management
T1 lesions may be managed by partial nephrectomy and this gives equivalent
oncological results to total radical nephrectomy. Partial nephrectomy may also be
performed when there is inadequate reserve in the remaining kidney.
For T2 lesions and above a radical nephrectomy is standard practice and this may
be performed via a laparoscopic or open approach. Preoperative embolisation is
not indicated nor is resection of uninvolved adrenal glands. During surgery early
venous control is mandatory to avoid shedding of tumour cells into the circulation.
Patients with completely resected disease do not benefit from adjuvant therapy
with either chemotherapy or biological agents. These should not be administered
outside the setting of clinical trials.
References
Lungberg B et al. EAU guidelines on renal cell carcinoma: The 2010 update.
European Urology 2010 (58): 398-406.
Next question
Save my notes
Question stats
A 35.8%
B 15.3%
C 36.2%
D 5.8%
E 6.8%
Search eMRCS
Search term Go
External links
+ Suggest a link
Dashboard
7
Question 11 of 63
gathered by dr. elbarky, for free, not intended for profit by anybody elsewhere
A 20 year old male notices a mild painful swelling of his right scrotum. He also
complains of abdominal pain. Clinically, the patient is found to have a swollen right
testicle. Apart from a supraclavicular node, there is no obvious lymphadenopathy.
What is the best course of action?
Administration of antibiotics
Next question
Scrotal swelling
Differential diagnosis
Management
Testicular malignancy is always treated with orchidectomy via an inguinal
approach. This allows high ligation of the testicular vessels and avoids
exposure of another lymphatic field to the tumour.
Torsion is commonest in young teenagers and the history in older children
can be difficult to elicit. Intermittent torsion is a recognised problem. The
treatment is prompt surgical exploration and testicular fixation. This can be
achieved using sutures or by placement of the testis in a Dartos pouch.
Varicoceles are usually managed conservatively. If there are concerns about
testicular function of infertility then surgery or radiological management can
gathered by dr. elbarky, for free, not intended for profit by anybody elsewhere
be considered.
Epididymal cysts can be excised using a scrotal approach
Hydroceles are managed differently in children where the underlying
pathology is a patent processus vaginalis and therefore an inguinal
approach is used in children so that the processus can be ligated. In adults
a scrotal approach is preferred and the hydrocele sac excised or plicated.
Next question
Save my notes
Question stats
A 11.6%
B 9.9%
C 15.2%
D 42.9%
E 20.4%
Search eMRCS
Search term Go
Question 12 of 63
gathered by dr. elbarky, for free, not intended for profit by anybody elsewhere
A 28 year old man presents with pain in the testis and scrotum. It began 10 hours
previously and has worsened during that time. On examination, he is pyrexial, the
testis is swollen and tender and there is an associated hydrocele. What is the most
likely diagnosis?
Testicular seminoma
Acute epididymo-orchitis
The onset is relatively slow for torsion and the presence of fever favors epididymo-
orchitis.
Next question
Testicular disorders
Testicular cancer
Testicular cancer is the most common malignancy in men aged 20-30 years.
Around 95% of cases of testicular cancer are germ-cell tumours. Germ cell
tumours may essentially be divided into:
Tumour
Tumour type Key features markers Pathology
Tumour
Tumour type Key features markers Pathology
(https://d2zgo9qer4wjf4.cloudfront.net/images_eMRCS/swb090b.jpg)
Image sourced from Wikipedia
(https://d2zgo9qer4wjf4.cloudfront.net
(http://en.wikipedia.org
/images_eMRCS/swb090b.jpg)
/wiki/Seminoma)
Features
A painless lump is the most common presenting symptom
Pain may also be present in a minority of men
Other possible features include hydrocele, gynaecomastia
Diagnosis
Ultrasound is first-line
CT scanning of the chest/ abdomen and pelvis is used for staging
Tumour markers (see above) should be measured
gathered by dr. elbarky, for free, not intended for profit by anybody elsewhere
Management
Orchidectomy (Inguinal approach)
Chemotherapy and radiotherapy may be given depending on staging
Abdominal lesions >1cm following chemotherapy may require
retroperitoneal lymph node dissection.
Benign disease
Epididymo-orchitis
Acute epididymitis is an acute inflammation of the epididymis, often involving the
testis and usually caused by bacterial infection.
Infection spreads from the urethra or bladder. In men <35 years, gonorrhoea
or chlamydia are the usual infections.
Amiodarone is a recognised non infective cause of epididymitis, which
resolves on stopping the drug.
Tenderness is usually confined to the epididymis, which may facilitate
differentiating it from torsion where pain usually affects the entire testis.
Testicular torsion
Twist of the spermatic cord resulting in testicular ischaemia and necrosis.
Most common in males aged between 10 and 30 (peak incidence 13-15
years)
Pain is usually severe and of sudden onset.
Cremasteric reflex is lost and elevation of the testis does not ease the pain.
Treatment is with surgical exploration. If a torted testis is identified then
both testis should be fixed as the condition of bell clapper testis is often
bilateral.
Hydrocele
Presents as a mass that transilluminates, usually possible to 'get above' it
on examination.
In younger men it should be investigated with USS to exclude tumour.
In children it may occur as a result of a patent processus vaginalis.
Treatment in adults is with a Lords or Jabouley procedure.
Treatment in children is with trans inguinal ligation of PPV.
Next question
gathered by dr. elbarky, for free, not intended for profit by anybody elsewhere
Save my notes
Question stats
A 11.5%
B 12.9%
C 6.6%
D 61.4%
E 7.6%
Search eMRCS
Search term Go
External links
+ Suggest a link
Dashboard
3
Question 13 of 63
gathered by dr. elbarky, for free, not intended for profit by anybody elsewhere
What is the most useful test for a 5 year old who has vesicoureteric reflux in whom
there are concerns about the potential of renal scarring?
Intravenous urogram
Renal CT scan
DMSA scan
Micturating cystourethrogram
Retrograde ureterogram
Next question
DMSA scan
Dimercaptosuccinic acid (DMSA) scintigraphy
DMSA localises to the renal cortex with little accumulation in the renal papilla and
medulla. It is useful for the identification of cortical defects and ectopic or
aberrant kidneys. It does not provide useful information on the ureter of collecting
system.
MAG 3 renogram
Mercaptoacetyle triglycine is an is extensively protein bound and is primarily
gathered by dr. elbarky, for free, not intended for profit by anybody elsewhere
secreted by tubular cells rather than filtered at the glomerulus. This makes it the
agent of choice for imaging the kidneys of patients with existing renal impairment
(where GFR is impaired).
PET/CT
This may be used to evaluate structurally indeterminate lesions in the staging of
malignancy.
References
Davis A et al. Investigating urinary tract infections in children. BMJ 2013
(346):35-37.
Next question
Save my notes
Question stats
A 8.4%
B 8.1%
C 54.4%
D 16%
E 13.2%
gathered by dr. elbarky, for free, not intended for profit by anybody elsewhere
Search eMRCS
Search term Go
External links
+ Suggest a link
Dashboard
10
11
12
13
14
Question 14 of 63
gathered by dr. elbarky, for free, not intended for profit by anybody elsewhere
A 56 year old man presents with urinary symptoms and on investigation is found to
have a 2cm nodule in the left lobe of the prostate. Imaging with whole body MRI
and pelvic CT/MRI demonstrates a likely cancer with no distant disease and no
nodal metastasis. What is the most appropriate course of action?
Robotic prostatectomy
Open prostatectomy
Transvesical prostatectomy
Prostatic biopsy
Next question
Prostate Cancer
Prostate Cancer
This is a common condition and up to 30,000 men are diagnosed with the
condition each year. Up to 9,000 will die in in the UK from the condition per year.
Diagnosis
Early prostate cancers have few symptoms.
Metastatic disease may present as bone pain.
Locally advanced disease may present as pelvic pain or with urinary symptoms.
Prostate specific antigen measurement
Digital rectal examination
Trans rectal USS (+/- biopsy)
MRI/ CT and bone scan for staging.
PSA Test
The normal upper limit for PSA is 4ng/ml. However, in this group will lie patients
with benign disease and some with localised prostate cancer. False positives may
gathered by dr. elbarky, for free, not intended for profit by anybody elsewhere
Pathology
95% adenocarcinoma
In situ malignancy is sometimes found in areas adjacent to cancer. Multiple
biopsies needed to call true in situ disease.
Often multifocal- 70% lie in the peripheral zone.
Graded using the Gleason grading system, two grades awarded 1 for most
dominant grade (on scale of 1-5) and 2 for second most dominant grade
(scale 1-5). The two added together give the Gleason score. Where 2 is best
prognosis and 10 the worst.
Lymphatic spread occurs first to the obturator nodes and local extra
prostatic spread to the seminal vesicles is associated with distant disease.
Treatment
Watch and wait- Elderly, multiple co-morbidities, low Gleason score
Radiotherapy (External)- Both potentially curative and palliative therapy
possible. However, radiation proctitis and rectal malignancy are late
problems. Brachytherapy is a modification allowing internal radiotherapy.
Surgery- Radical prostatectomy. Surgical removal of the prostate is the
standard treatment for localised disease. The robot is being used
increasingly for this procedure. As well as the prostate the obturator nodes
are also removed to complement the staging process. Erectile dysfunction
is a common side effect. Survival may be better than with radiotherapy (see
references).
Hormonal therapy- Testosterone stimulates prostate tissue and prostatic
cancers usually show some degree of testosterone dependence. 95% of
testosterone is derived from the testis and bilateral orchidectomy may be
used for this reason. Pharmacological alternatives include LHRH analogues
and anti androgens (which may be given in combination).
In the UK the National Institute for Clinical Excellence (NICE) suggests that
active surveillance is the preferred option for low risk men. It is particularly
suitable for men with clinical stage T1c, Gleason score 3+3 and PSA density
< 0.15 ng/ml/ml who have cancer in less than 50% of their biopsy cores,
with < 10 mm of any core involved.
Next question
Save my notes
Question stats
A 18.9%
B 13.1%
C 18.7%
D 41.5%
E 7.8%
Search eMRCS
Search term Go
Question 15 of 63
gathered by dr. elbarky, for free, not intended for profit by anybody elsewhere
A 22 year old man presents with an aching pain and discomfort in his right testicle.
He has felt systemically unwell for the preceding 48 hours. On examination, there
is tenderness of the right testicle. He has an exaggerated cremasteric reflex. What
is the correct course of action?
Administration of antibiotics
Next question
Testicular disorders
Testicular cancer
Testicular cancer is the most common malignancy in men aged 20-30 years.
Around 95% of cases of testicular cancer are germ-cell tumours. Germ cell
tumours may essentially be divided into:
Tumour
Tumour type Key features markers Pathology
Tumour
Tumour type Key features markers Pathology
(https://d2zgo9qer4wjf4.cloudfront.net/images_eMRCS/swb090b.jpg)
Image sourced from Wikipedia
(https://d2zgo9qer4wjf4.cloudfront.net
(http://en.wikipedia.org
/images_eMRCS/swb090b.jpg)
/wiki/Seminoma)
Features
A painless lump is the most common presenting symptom
Pain may also be present in a minority of men
Other possible features include hydrocele, gynaecomastia
Diagnosis
Ultrasound is first-line
CT scanning of the chest/ abdomen and pelvis is used for staging
Tumour markers (see above) should be measured
gathered by dr. elbarky, for free, not intended for profit by anybody elsewhere
Management
Orchidectomy (Inguinal approach)
Chemotherapy and radiotherapy may be given depending on staging
Abdominal lesions >1cm following chemotherapy may require
retroperitoneal lymph node dissection.
Benign disease
Epididymo-orchitis
Acute epididymitis is an acute inflammation of the epididymis, often involving the
testis and usually caused by bacterial infection.
Infection spreads from the urethra or bladder. In men <35 years, gonorrhoea
or chlamydia are the usual infections.
Amiodarone is a recognised non infective cause of epididymitis, which
resolves on stopping the drug.
Tenderness is usually confined to the epididymis, which may facilitate
differentiating it from torsion where pain usually affects the entire testis.
Testicular torsion
Twist of the spermatic cord resulting in testicular ischaemia and necrosis.
Most common in males aged between 10 and 30 (peak incidence 13-15
years)
Pain is usually severe and of sudden onset.
Cremasteric reflex is lost and elevation of the testis does not ease the pain.
Treatment is with surgical exploration. If a torted testis is identified then
both testis should be fixed as the condition of bell clapper testis is often
bilateral.
Hydrocele
Presents as a mass that transilluminates, usually possible to 'get above' it
on examination.
In younger men it should be investigated with USS to exclude tumour.
In children it may occur as a result of a patent processus vaginalis.
Treatment in adults is with a Lords or Jabouley procedure.
Treatment in children is with trans inguinal ligation of PPV.
Next question
gathered by dr. elbarky, for free, not intended for profit by anybody elsewhere
Save my notes
Question stats
A 21.2%
B 12.2%
C 53.6%
D 6.2%
E 6.8%
Search eMRCS
Search term Go
External links
+ Suggest a link
Dashboard
3
Question 16 of 63
gathered by dr. elbarky, for free, not intended for profit by anybody elsewhere
A 3 month old boy is brought to the clinic by his mother who has noticed a swelling
in the right hemiscrotum. On examination, there is a firm mass affecting the right
spermatic cord distally, the testis is felt separately from it. What is the most likely
diagnosis?
Rhabdomyosarcoma
Hydrocele
Next question
Testicular disorders
Testicular cancer
Testicular cancer is the most common malignancy in men aged 20-30 years.
Around 95% of cases of testicular cancer are germ-cell tumours. Germ cell
tumours may essentially be divided into:
Tumour
Tumour type Key features markers Pathology
gathered by dr. elbarky, for free, not intended for profit by anybody elsewhere
(https://d2zgo9qer4wjf4.cloudfront.net/images_eMRCS/swb090b.jpg)
Image sourced from Wikipedia
(https://d2zgo9qer4wjf4.cloudfront.net
(http://en.wikipedia.org
/images_eMRCS/swb090b.jpg)
/wiki/Seminoma)
Features
A painless lump is the most common presenting symptom
Pain may also be present in a minority of men
Other possible features include hydrocele, gynaecomastia
Diagnosis
Ultrasound is first-line
CT scanning of the chest/ abdomen and pelvis is used for staging
Tumour markers (see above) should be measured
gathered by dr. elbarky, for free, not intended for profit by anybody elsewhere
Management
Orchidectomy (Inguinal approach)
Chemotherapy and radiotherapy may be given depending on staging
Abdominal lesions >1cm following chemotherapy may require
retroperitoneal lymph node dissection.
Benign disease
Epididymo-orchitis
Acute epididymitis is an acute inflammation of the epididymis, often involving the
testis and usually caused by bacterial infection.
Infection spreads from the urethra or bladder. In men <35 years, gonorrhoea
or chlamydia are the usual infections.
Amiodarone is a recognised non infective cause of epididymitis, which
resolves on stopping the drug.
Tenderness is usually confined to the epididymis, which may facilitate
differentiating it from torsion where pain usually affects the entire testis.
Testicular torsion
Twist of the spermatic cord resulting in testicular ischaemia and necrosis.
Most common in males aged between 10 and 30 (peak incidence 13-15
years)
Pain is usually severe and of sudden onset.
Cremasteric reflex is lost and elevation of the testis does not ease the pain.
Treatment is with surgical exploration. If a torted testis is identified then
both testis should be fixed as the condition of bell clapper testis is often
bilateral.
Hydrocele
Presents as a mass that transilluminates, usually possible to 'get above' it
on examination.
In younger men it should be investigated with USS to exclude tumour.
In children it may occur as a result of a patent processus vaginalis.
Treatment in adults is with a Lords or Jabouley procedure.
Treatment in children is with trans inguinal ligation of PPV.
Next question
gathered by dr. elbarky, for free, not intended for profit by anybody elsewhere
Save my notes
Question stats
A 21.6%
B 32.5%
C 20.2%
D 13%
E 12.7%
Search eMRCS
Search term Go
External links
+ Suggest a link
Dashboard
3
Question 17 of 63
gathered by dr. elbarky, for free, not intended for profit by anybody elsewhere
Abdominal ultrasound
Scrotal ultrasound
Left orchidectomy
Discharge
Next question
Renal tumours
Investigation
Many cases will present as haematuria and be discovered during diagnostic work
up. Benign renal tumours are rare, so renal masses should be investigated with
multislice CT scanning. Some units will add an arterial and venous phase to the
scan to demonstrate vascularity and evidence of caval ingrowth.
CT scanning of the chest and abdomen to detect distant disease should also be
undertaken.
Management
T1 lesions may be managed by partial nephrectomy and this gives equivalent
oncological results to total radical nephrectomy. Partial nephrectomy may also be
performed when there is inadequate reserve in the remaining kidney.
For T2 lesions and above a radical nephrectomy is standard practice and this may
be performed via a laparoscopic or open approach. Preoperative embolisation is
not indicated nor is resection of uninvolved adrenal glands. During surgery early
venous control is mandatory to avoid shedding of tumour cells into the circulation.
Patients with completely resected disease do not benefit from adjuvant therapy
with either chemotherapy or biological agents. These should not be administered
outside the setting of clinical trials.
References
Lungberg B et al. EAU guidelines on renal cell carcinoma: The 2010 update.
European Urology 2010 (58): 398-406.
Next question
Display my notes on this topic
gathered by dr. elbarky, for free, not intended for profit by anybody elsewhere
Save my notes
Question stats
A 13.1%
B 48.4%
C 21.9%
D 6.9%
E 9.6%
Search eMRCS
Search term Go
External links
+ Suggest a link
Dashboard
5
Question 18 of 63
gathered by dr. elbarky, for free, not intended for profit by anybody elsewhere
A 47-year-old woman presents with loin pain and haematuria. Urine dipstick
demonstrates:
Blood ++++
Nitrites POS
Leucocytes +++
Protein ++
Xanthine
Calcium oxalate
Struvite
Cystine
Urate
The table below summarises the appearance of different types of renal stone on
x-ray
*stag-horn calculi involve the renal pelvis and extend into at least 2 calyces. They
develop in alkaline urine and are composed of struvite (ammonium magnesium
phosphate, triple phosphate). Ureaplasma urealyticum and Proteus infections
predispose to their formation
Next question
Save my notes
Question stats
A 5.8%
B 11.2%
68.7% of users answered this question correctly
gathered by dr. elbarky, for free, not intended for profit by anybody elsewhere
C 68.7%
D 7.4%
E 7% Search eMRCS
Search term Go
External links
+ Suggest a link
Dashboard
10
11
12
13
14
15
Question 19 of 63
gathered by dr. elbarky, for free, not intended for profit by anybody elsewhere
A 32 year old male presents with a swollen, painful right scrotum after being
kicked in the groin area 1 hour ago. There is a painful swelling of the right scrotum
and the underlying testis cannot be easily palpated. What is the most likely
explanation?
Hydrocele
Haematocele
Epididymo-orchitis
Orchitis
Epididymal cyst
Acute haematocele: tense, tender and non transilluminating mass post trauma. A
chronic haematoma causes a blood clot to surround the testis. The blood clot
hardens and contracts causing a hard mass which may be indistinguishable from
a tumour. Therefore the testis will need surgical exploration.
Next question
Scrotal swelling
Differential diagnosis
Management
Testicular malignancy is always treated with orchidectomy via an inguinal
approach. This allows high ligation of the testicular vessels and avoids
exposure of another lymphatic field to the tumour.
Torsion is commonest in young teenagers and the history in older children
can be difficult to elicit. Intermittent torsion is a recognised problem. The
treatment is prompt surgical exploration and testicular fixation. This can be
achieved using sutures or by placement of the testis in a Dartos pouch.
Varicoceles are usually managed conservatively. If there are concerns about
testicular function of infertility then surgery or radiological management can
gathered by dr. elbarky, for free, not intended for profit by anybody elsewhere
be considered.
Epididymal cysts can be excised using a scrotal approach
Hydroceles are managed differently in children where the underlying
pathology is a patent processus vaginalis and therefore an inguinal
approach is used in children so that the processus can be ligated. In adults
a scrotal approach is preferred and the hydrocele sac excised or plicated.
Next question
Save my notes
Question stats
A 6.5%
B 80.3%
C 4.7%
D 4.4%
E 4.1%
Search eMRCS
Search term Go
Question 23 of 63
gathered by dr. elbarky, for free, not intended for profit by anybody elsewhere
A 56 year old man is admitted with severe loin to groin pain associated with
haematuria. He was well until 1 week ago when he was unwell with diarrhoea and
vomiting. What is the most likely cause?
Ureteric calculus
Renal cancer
Pyelonephritis
Prostatitis
Prostate cancer
Next question
Renal stones
Type of Percentage of
stones Features all calculi
Type of Percentage of
stones Features all calculi
oxalate causes)
Hyperoxaluria may also increase risk
Hypocitraturia increases risk because citrate
forms complexes with calcium making it more
soluble
Stones are radio-opaque (though less than
calcium phosphate stones)
Hyperuricosuria may cause uric acid stones to
which calcium oxalate binds
Next question
Save my notes
Question stats
A 58.9%
B 5.8%
C 25%
D 5.9%
E 4.4%
Search eMRCS
Question 20 of 63
gathered by dr. elbarky, for free, not intended for profit by anybody elsewhere
A 56 year old man is admitted with acute retention of urine. He has had a recent
urinary tract infection. An USS shows bilateral hydronephrosis. What is the best
course of action?
Urethral catheter
Bilateral nephrostomy
Suprapubic catheter
Establishing bladder drainage will often correct the situation. These patients often
have a significant diuresis with associated electrolyte disturbance. The urethral
route should be tried first.
Next question
Hydronephrosis
Causes of hydronephrosis
Unilateral: PACT
Pelvic-ureteric obstruction (congenital or acquired)
Aberrant renal vessels
Calculi
Tumours of renal pelvis
Bilateral: SUPER
Stenosis of the urethra
Urethral valve
Prostatic enlargement
Extensive bladder tumour
Retro-peritoneal fibrosis
gathered by dr. elbarky, for free, not intended for profit by anybody elsewhere
Investigation
USS- identifies presence of hydronephrosis and can assess the kidneys
IVU- assess the position of the obstruction
Antegrade or retrograde pyelography- allows treatment
If renal colic suspected: non contrast CT scan (majority of stones are
detected this way)
Management
Remove the obstruction and drainage of urine
Acute upper urinary tract obstruction: Nephrostomy tube
Chronic upper urinary tract obstruction: Ureteric stent or a pyeloplasty
Next question
Save my notes
Question stats
A 10.2%
B 11.5%
C 50.1%
D 15.7%
E 12.5%
Search eMRCS
Question 21 of 63
gathered by dr. elbarky, for free, not intended for profit by anybody elsewhere
A 25 year old man is admitted with left sided loin pain that radiates to his groin.
His investigations demonstrate a 9mm left sided calculus within the proximal
ureter. His renal function is normal. What is the most appropriate course of action?
For ureteric stones with a maximum diameter of less than 10mm the first-line
treatment is extracorporeal shock wave lithotripsy (ESWL). If ESWL fails or if the
stone is impacted in the upper ureter then ureteroscopy can be performed.
Next question
Management
Most renal stones measuring less than 5mm in maximum diameter will typically
pass within 4 weeks of symptom onset(1)(2). More urgent treatment is indicated in
the presence of ureteric obstruction, renal developmental abnormalities such as
horseshoe kidney and previous renal transplant. Ureteric obstruction together with
infection is a surgical emergency and the system must be decompressed. Options
include nephrostomy tube placement or ureteric stent placement via cystoscopy.
gathered by dr. elbarky, for free, not intended for profit by anybody elsewhere
In the non-emergency setting the preferred options for treatment of stone disease
include extracorporeal shock wave lithotripsy (ESWL), percutaneous
nephrolithotomy (PCNL) and ureteroscopy (URS). These minimally invasive options
are the most popular first line treatments. Open surgery remains an option in a few
selected cases.
Ureteroscopy
A ureteroscope is passed retrograde via the urethra and bladder, into the ureter and
renal pelvis. Laser or pneumatic fragmentation (lithoclasty) is performed using the
ureteroscope and stone fragments are extracted(3). In some cases a stent is left in
situ after the procedure.
Percutaneous nephrolithotomy
In this procedure percutaneous access is gained to the renal collecting system.
Once access is achieved, endoscopic intra-corporeal lithotripsy, lithoclasty or laser
stone fragmentation is performed and stone fragments removed.
Therapeutic selection
Renal stones
Ureteric stones
5-10mm ESWL
gathered by dr. elbarky, for free, not intended for profit by anybody elsewhere
10-20mm Ureteroscopy
Reference
1. Bultitude M, Rees J. Management of renal colic. BMJ. 2012;345(7872):18.
2. Shah TT, Gao C, Peters M, Manning T, Cashman S, Nambiar A, et al. Factors
associated with spontaneous stone passage in a contemporary cohort of patients
presenting with acute ureteric colic: results from the Multi-centre cohort study
evaluating the role of Inflammatory Markers In patients presenting with acute
ureteric . BJU Int. 2019 Apr.
3. BAUS. Ureteroscopy (Telescopic surgery for stone removal) (Internet). BAUS
patient information leaflet. 2017 (cited 2019 Jun 27). p. 16. Available from:
https://www.baus.org.uk/userfiles/pages/files/Patients/Leaflets/Ureteroscopy for
stone.pdf
4. BAUS. Percutaneous nephrolithotomy (keyhole surgery for kidney stones). BAUS
patient information leaflet. 2017. p. 16. Available from: https://www.baus.org.uk
/userfiles/pages/files/Patients/Leaflets/PCNL.pdf
5. NICE. Renal and ureteric stones: assessment and management. NICE
Guide[NG118]. 2019;(January):112.
Next question
Save my notes
Question stats
A 14.6%
B 54.2%
C 11.7%
D 14.4%
E 5.1%
Question 22 of 63
gathered by dr. elbarky, for free, not intended for profit by anybody elsewhere
Radiotherapy alone
In men with metastatic bone lesions from prostate cancer, the best outcomes are
achieved with androgen suppression. Radiotherapy can also produced marked
palliation. A 2010 Cochrane review has clearly demonstrated added benefit, in
terms of symptom control, from the addition of a bisphosphonate.
Next question
Prostate Cancer
Prostate Cancer
This is a common condition and up to 30,000 men are diagnosed with the
condition each year. Up to 9,000 will die in in the UK from the condition per year.
Diagnosis
Early prostate cancers have few symptoms.
Metastatic disease may present as bone pain.
Locally advanced disease may present as pelvic pain or with urinary symptoms.
Prostate specific antigen measurement
Digital rectal examination
Trans rectal USS (+/- biopsy)
MRI/ CT and bone scan for staging.
gathered by dr. elbarky, for free, not intended for profit by anybody elsewhere
PSA Test
The normal upper limit for PSA is 4ng/ml. However, in this group will lie patients
with benign disease and some with localised prostate cancer. False positives may
be due to prostatitis, UTI, BPH, vigorous DRE.
The percentage of free: total PSA may help to distinguish benign disease from
cancer. Values of <20% are suggestive of cancer and biopsy is advised.
Pathology
95% adenocarcinoma
In situ malignancy is sometimes found in areas adjacent to cancer. Multiple
biopsies needed to call true in situ disease.
Often multifocal- 70% lie in the peripheral zone.
Graded using the Gleason grading system, two grades awarded 1 for most
dominant grade (on scale of 1-5) and 2 for second most dominant grade
(scale 1-5). The two added together give the Gleason score. Where 2 is best
prognosis and 10 the worst.
Lymphatic spread occurs first to the obturator nodes and local extra
prostatic spread to the seminal vesicles is associated with distant disease.
Treatment
Watch and wait- Elderly, multiple co-morbidities, low Gleason score
Radiotherapy (External)- Both potentially curative and palliative therapy
possible. However, radiation proctitis and rectal malignancy are late
problems. Brachytherapy is a modification allowing internal radiotherapy.
Surgery- Radical prostatectomy. Surgical removal of the prostate is the
standard treatment for localised disease. The robot is being used
increasingly for this procedure. As well as the prostate the obturator nodes
are also removed to complement the staging process. Erectile dysfunction
is a common side effect. Survival may be better than with radiotherapy (see
references).
Hormonal therapy- Testosterone stimulates prostate tissue and prostatic
cancers usually show some degree of testosterone dependence. 95% of
testosterone is derived from the testis and bilateral orchidectomy may be
used for this reason. Pharmacological alternatives include LHRH analogues
and anti androgens (which may be given in combination).
In the UK the National Institute for Clinical Excellence (NICE) suggests that
active surveillance is the preferred option for low risk men. It is particularly
suitable for men with clinical stage T1c, Gleason score 3+3 and PSA density
< 0.15 ng/ml/ml who have cancer in less than 50% of their biopsy cores,
with < 10 mm of any core involved.
References
1. Prostate cancer pathway. NICE.(http://guidance.nice.org.uk/IPG424)
2. Sooriakumaran P et al. Comparative effectiveness of radical prostatectomy and
radiotherapy in prostate cancer: observational study of mortality outcomes. BMJ
2014 (348):13. This study shows that in men with localised disease survival was
greater in those offered surgery.
Next question
Save my notes
Question stats
A 5.5%
B 5.1%
C 20.5%
D 57.7%
E 11.2%
Search eMRCS
Search term Go
Question 24 of 63
gathered by dr. elbarky, for free, not intended for profit by anybody elsewhere
Radical cystectomy
Palliative radiotherapy
Intravesical BCG
Intravesical mitomycin C
Intravesical cisplatin
Next question
Bladder cancer
Bladder cancer is the second most common urological cancer. It most commonly
affects males aged between 50 and 80 years of age. Those who are current, or
previous (within 20 years), smokers have a 2-5 fold increased risk of the disease.
Exposure to hydrocarbons such as 2-Naphthylamine increases the risk. Although
rare in the UK, chronic bladder inflammation arising from Schistosomiasis
infection remains a common cause of squamous cell carcinomas, in those
countries where the disease is endemic.
Benign tumours
Benign tumours of the bladder including inverted urothelial papilloma and
nephrogenic adenoma are uncommon.
Bladder malignancies
gathered by dr. elbarky, for free, not intended for profit by anybody elsewhere
TNM Staging
Stage Description
T0 No evidence of tumour
N0 No nodal disease
M0 No distant metastasis
M1 Distant disease
Presentation
Most patients (85%) will present with painless, macroscopic haematuria. In those
patients with incidental microscopic haematuria, up to 10% of females aged over
50 will be found to have a malignancy (once infection excluded).
Staging
Most will undergo a cystoscopy and biopsies or TURBT, this provides histological
diagnosis and information relating to depth of invasion. Locoregional spread is
best determined using pelvic MRI and distant disease CT scanning. Nodes of
uncertain significance may be investigated using PET CT.
Treatment
Those with superficial lesions may be managed using TURBT in isolation. Those
with recurrences or higher grade/ risk on histology may be offered intravesical
chemotherapy. Those with T2 disease are usually offered either surgery (radical
cystectomy and ileal conduit) or radical radiotherapy.
Prognosis
T1 90%
T2 60%
T3 35%
T4a 10-25%
Next question
Save my notes
gathered by dr. elbarky, for free, not intended for profit by anybody elsewhere
Question stats
A 51.2%
B 5.8%
C 27%
D 9.5%
E 6.4%
Search eMRCS
Search term Go
External links
+ Suggest a link
Dashboard
8
Question 25 of 63
gathered by dr. elbarky, for free, not intended for profit by anybody elsewhere
A 56 year old lady reports incontinence mainly when walking the dog. A bladder
diary is inconclusive. What is the most appropriate investigation?
Intravenous urography
Urodynamic studies
Flexible cystoscopy
Micturating cystourethrogram
Rigid cystoscopy
Urodynamic studies are indicated when there is diagnostic uncertainty or plans for
surgery.
Next question
Urinary incontinence
Involuntary passage of urine. Most cases are female (80%). It has a prevalence of
11% in those aged greater than 65 years. The commonest variants include:
Stress urinary incontinence (50%)
Urge incontinence (15%)
Mixed (35%)
Males
Males may also suffer from incontinence although it is a much rarer condition in
men. A number of anatomical factors contribute to this. Males have 2 powerful
sphincters; one at the bladder neck and the other in the urethra. Damage to the
bladder neck mechanism is a factor in causing retrograde ejaculation following
prostatectomy. The short segment of urethra passing through the urogenital
diaphragm consists of striated muscle fibres (the external urethral sphincter) and
smooth muscle capable of more sustained contraction. It is the latter mechanism
that maintains continence following prostatectomy.
gathered by dr. elbarky, for free, not intended for profit by anybody elsewhere
Females
The sphincter complex at the level of bladder neck is poorly developed in females.
As a result the external sphincter complex is functionally more important, its
composition being similar to that of males. Innervation is via the pudendal nerve
and the neuropathy that may accompany obstetric events may compromise this
and lead to stress urinary incontinence.
Innervation
Somatic innervation to the bladder is via the pudendal, hypogastric and pelvic
nerves. Autonomic nerves travel in these nerve fibres too. Bladder filling leads to
detrusor relaxation (sympathetic) coupled with sphincter contraction. The
parasympathetic system causes detrusor contraction and sphincter relaxation.
Overall control of micturition is centrally mediated via centres in the Pons.
Urethral mobility:
Pressure not transmitted appropriately to the urethra resulting in involuntary
passage of urine during episodes of raised intra-abdominal pressure.
Sphincter dysfunction:
Sphincter fails to adapt to compress urethra resulting in involuntary passage of
urine. When the sphincter completely fails there is often to continuous passage of
urine.
Urge incontinence
In these patients there is sense of urgency followed by incontinence. The detrusor
muscle in these patients is unstable and urodynamic investigation will
demonstrate overactivity of the detrusor muscle at inappropriate times (e.g.
Bladder filling). Urgency may be seen in patients with overt neurological disorders
and those without. The pathophysiology is not well understood but poor central
and peripheral co-ordination of the events surrounding bladder filling are the main
processes.
Assessment
Careful history and examination including vaginal examination for cystocele.
Bladder diary for at least 3 days
Consider flow cystometry if unclear symptomatology or surgery considered and
diagnosis is unclear.
Exclusion of other organic disease (e.g. Stones, UTI, Cancer)
Management
Conservative measures should be tried first; Stress urinary incontinence or mixed
gathered by dr. elbarky, for free, not intended for profit by anybody elsewhere
symptoms should undergo 3 months of pelvic floor exercise. Over active bladder
should have 6 weeks of bladder retraining.
Drug therapy for women with overactive bladder should be offered oxybutynin (or
solifenacin if elderly) if conservative measures fail.
In women with detrusor instability who fail non operative therapy a trial of sacral
neuromodulation may be considered, with conversion to permanent implant if
good response. Augmentation cystoplasty is an alternative but will involve long
term intermittent self catheterisation.
In women with stress urinary incontinence a urethral sling type procedure may be
undertaken. Where cystocele is present in association with incontinence it should
be repaired particularly if it lies at the introitus.
NICE guidelines
Initial assessment urinary incontinence should be classified as stress/urge
/mixed.
At least 3/7 bladder diary if unable to classify easily.
Start conservative treatment before urodynamic studies if a diagnosis is
obvious from the history
Urodynamic studies if plans for surgery.
Stress incontinence: Pelvic floor exercises 3/12, if fails consider surgery.
Urge incontinence: Bladder training >6/52, if fails for oxybutynin
(antimuscarinic drugs) then sacral nerve stimulation.
Pelvic floor exercises offered to all women in their 1st pregnancy.
Next question
Save my notes
Question stats
A 5.2%
B 75.2%
C 5.4%
Question 26 of 63
gathered by dr. elbarky, for free, not intended for profit by anybody elsewhere
Which of the following procedures represents the optimal operative procedure for
testicular cancer?
Lords procedure
Testicular tumours metastasise to Para aortic nodes and thus an inguinal rather
than scrotal approach should be used. There are two main operations that are
termed Lords procedure; one is for fissure in ano and the other is a procedure for
hydrocele.
Next question
Testicular disorders
Testicular cancer
Testicular cancer is the most common malignancy in men aged 20-30 years.
Around 95% of cases of testicular cancer are germ-cell tumours. Germ cell
tumours may essentially be divided into:
Tumour
Tumour type Key features markers Pathology
Tumour
Tumour type Key features markers Pathology
(https://d2zgo9qer4wjf4.cloudfront.net/images_eMRCS/swb090b.jpg)
Image sourced from Wikipedia
(https://d2zgo9qer4wjf4.cloudfront.net
(http://en.wikipedia.org
/images_eMRCS/swb090b.jpg)
/wiki/Seminoma)
Features
A painless lump is the most common presenting symptom
Pain may also be present in a minority of men
Other possible features include hydrocele, gynaecomastia
Diagnosis
Ultrasound is first-line
CT scanning of the chest/ abdomen and pelvis is used for staging
Tumour markers (see above) should be measured
gathered by dr. elbarky, for free, not intended for profit by anybody elsewhere
Management
Orchidectomy (Inguinal approach)
Chemotherapy and radiotherapy may be given depending on staging
Abdominal lesions >1cm following chemotherapy may require
retroperitoneal lymph node dissection.
Benign disease
Epididymo-orchitis
Acute epididymitis is an acute inflammation of the epididymis, often involving the
testis and usually caused by bacterial infection.
Infection spreads from the urethra or bladder. In men <35 years, gonorrhoea
or chlamydia are the usual infections.
Amiodarone is a recognised non infective cause of epididymitis, which
resolves on stopping the drug.
Tenderness is usually confined to the epididymis, which may facilitate
differentiating it from torsion where pain usually affects the entire testis.
Testicular torsion
Twist of the spermatic cord resulting in testicular ischaemia and necrosis.
Most common in males aged between 10 and 30 (peak incidence 13-15
years)
Pain is usually severe and of sudden onset.
Cremasteric reflex is lost and elevation of the testis does not ease the pain.
Treatment is with surgical exploration. If a torted testis is identified then
both testis should be fixed as the condition of bell clapper testis is often
bilateral.
Hydrocele
Presents as a mass that transilluminates, usually possible to 'get above' it
on examination.
In younger men it should be investigated with USS to exclude tumour.
In children it may occur as a result of a patent processus vaginalis.
Treatment in adults is with a Lords or Jabouley procedure.
Treatment in children is with trans inguinal ligation of PPV.
Next question
gathered by dr. elbarky, for free, not intended for profit by anybody elsewhere
Save my notes
Question stats
A 6.9%
B 12.5%
C 67.3%
D 8.1%
E 5.3%
Search eMRCS
Search term Go
External links
+ Suggest a link
Dashboard
3
Question 27 of 63
gathered by dr. elbarky, for free, not intended for profit by anybody elsewhere
A 22 year old man is involved in a road traffic accident. He is found to have a pelvic
fracture. While on the ward the nursing staff report that he is complaining of lower
abdominal pain. On examination, you find a distended tender bladder and blood at
the urethral meatus. What is the best management?
Suprapubic catheter
This patient has possible urethral injury based on the history. Urethral
catheterisation is contraindicated in this situation.
Next question
Types of injury
Urethral injury Mainly in males
i.Bulbar rupture
- most common
- straddle type injury e.g. bicycles
- triad signs: urinary retention, perineal haematoma,
blood at the meatus
ii. Membranous rupture
- can be extra or intraperitoneal
- commonly due to pelvic fracture
- Penile or perineal oedema/ hematoma
- PR: prostate displaced upwards (beware co-existing
retroperitoneal haematomas as they may make
examination difficult)
Next question
Save my notes
gathered by dr. elbarky, for free, not intended for profit by anybody elsewhere
Question stats
A 7.2%
B 68.9%
C 11.1%
D 6.7%
E 6.2%
Search eMRCS
Search term Go
External links
+ Suggest a link
Dashboard
8
Question 28 of 63
gathered by dr. elbarky, for free, not intended for profit by anybody elsewhere
A 72 year old man presents with lower urinary tract symptoms. On digital rectal
examination, benign prostatic hyperplasia is suspected. Which of the following
treatments is associated with a reduction in the risk of urinary retention?
Alfuzosin
Finasteride
Prazosin
Tamsulosin
Terazosin
Reference
McConnell J et al. The effect of finasteride on the risk of urinary retention and the
need for surgical intervention amongst men with benign prostatic hyperplasia. N
Engl J Med 338:557-563
Next question
Benign prostatic hyperplasia occurs via an increase in the epithelial and stromal
cell numbers in the peri-urethral zone of the prostate. BPH is very common and
90% of men aged over 80 will have at least microscopic evidence of benign
prostatic hyperplasia. The causes of BPH are still not well understood, but the
importance of androgens remains appreciated even if the exact role by which they
induce BPH is elusive.
Presentation
The vast majority of men will present with lower urinary tract symptoms. These will
typically be:
gathered by dr. elbarky, for free, not intended for profit by anybody elsewhere
Poor flow
Nocturia
Hesitancy
Incomplete and double voiding
Terminal dribbling
Urgency
Incontinence
Investigation
Digital rectal examination to assess prostatic size and morphology.
Urine dipstick for infections and haematuria.
Uroflowmetry (a flow rate of >15ml/second helps to exclude BOO)
Bladder pressure studies may help identify detrusor failure and whilst may
not form part of first line investigations should be included in those with
atypical symptoms and prior to redo surgery.
Bladder scanning to demonstrate residual volumes. USS if high pressure
chronic retention.
Management
Lifestyle changes such as stopping smoking and altering fluid intake may
help those with mild symptoms.
Medical therapy includes alpha blockers and 5 α reductase inhibitors. The
former work quickly on receptor zones located at the bladder neck.
Cardiovascular side effects are well documented. The latter work on
testosterone metabolising enzymes. Although they have a slower onset of
action, the 5 α reductase inhibitors may prevent acute urinary retention.
Surgical therapy includes transurethral resection of the prostate and is the
treatment of choice in those with severe symptoms and those who fail to
respond to medical therapy. More tailored bladder neck incision procedures
may be considered in those with small prostates. Retrograde ejaculation
may occur following surgery. The change in the type of irrigation solutions
used has helped to minimise the TURP syndrome of electrolyte
disturbances.
Next question
Save my notes
Question stats
A 6.4%
B 51.5%
C 6.5%
D 29.6%
E 6.1%
Search eMRCS
Search term Go
External links
+ Suggest a link
Dashboard
7
Question 29 of 63
gathered by dr. elbarky, for free, not intended for profit by anybody elsewhere
A 17 year old man is referred to the urology clinic. As a child he was diagnosed as
having a right sided PUJ obstruction. However, he was lost to follow up. Over the
past 7 months he has been complaining of recurrent episodes of right loin pain. A
CT scan shows considerable renal scarring. What is the most useful investigation?
DMSA scan
MAG 3 renogram
CT KUB
Renal USS
In patients with long standing PUJ obstruction and renal scarring the main
diagnostic question is whether the individual has sufficient renal function to
consider a pyeloplasty or whether a primary nephrectomy is preferable. Since the
CT has demonstrated scarring there is no use in obtaining a DMSA scan. Of the
investigations listed both a DMSA and MAG 3 renogram will allow assessment of
renal function. However, MAG 3 is superior in the assessment of renal function in
damaged kidneys (as it is subjected to tubular secretion).
Next question
DMSA scan
Dimercaptosuccinic acid (DMSA) scintigraphy
DMSA localises to the renal cortex with little accumulation in the renal papilla and
medulla. It is useful for the identification of cortical defects and ectopic or
aberrant kidneys. It does not provide useful information on the ureter of collecting
system.
Diethylene-triamine-penta-acetic acid (DTPA)
This is primarily a glomerular filtration agent. It is most useful for the assessment
of renal function. Because it is filtered at the level of the glomerulus it provides
gathered by dr. elbarky, for free, not intended for profit by anybody elsewhere
useful information about the GFR. Image quality may be degraded in patients with
chronic renal impairment and derangement of GFR.
MAG 3 renogram
Mercaptoacetyle triglycine is an is extensively protein bound and is primarily
secreted by tubular cells rather than filtered at the glomerulus. This makes it the
agent of choice for imaging the kidneys of patients with existing renal impairment
(where GFR is impaired).
PET/CT
This may be used to evaluate structurally indeterminate lesions in the staging of
malignancy.
References
Davis A et al. Investigating urinary tract infections in children. BMJ 2013
(346):35-37.
Next question
Save my notes
Question stats
A 45.4%
gathered by dr. elbarky, for free, not intended for profit by anybody elsewhere
B 39.5%
C 4.9%
D 5.2%
E 4.9%
Search eMRCS
Search term Go
External links
+ Suggest a link
Dashboard
10
Question 30 of 63
gathered by dr. elbarky, for free, not intended for profit by anybody elsewhere
Routine renal biopsy should not be performed in cases for nephrectomy. Most
cases of malignancy can be accurately classified on imaging.
Next question
Renal tumours
Investigation
Many cases will present as haematuria and be discovered during diagnostic work
up. Benign renal tumours are rare, so renal masses should be investigated with
multislice CT scanning. Some units will add an arterial and venous phase to the
scan to demonstrate vascularity and evidence of caval ingrowth.
CT scanning of the chest and abdomen to detect distant disease should also be
undertaken.
Management
T1 lesions may be managed by partial nephrectomy and this gives equivalent
oncological results to total radical nephrectomy. Partial nephrectomy may also be
performed when there is inadequate reserve in the remaining kidney.
For T2 lesions and above a radical nephrectomy is standard practice and this may
be performed via a laparoscopic or open approach. Preoperative embolisation is
not indicated nor is resection of uninvolved adrenal glands. During surgery early
venous control is mandatory to avoid shedding of tumour cells into the circulation.
Patients with completely resected disease do not benefit from adjuvant therapy
with either chemotherapy or biological agents. These should not be administered
outside the setting of clinical trials.
References
Lungberg B et al. EAU guidelines on renal cell carcinoma: The 2010 update.
European Urology 2010 (58): 398-406.
Next question
Display my notes on this topic
gathered by dr. elbarky, for free, not intended for profit by anybody elsewhere
Save my notes
Question stats
A 11.7%
B 49.2%
C 8.3%
D 21%
E 9.8%
Search eMRCS
Search term Go
External links
+ Suggest a link
Dashboard
5
Question 31 of 63
gathered by dr. elbarky, for free, not intended for profit by anybody elsewhere
Cystine stone
Struvite
Chemotherapy and cell death can increase uric acid levels. In this acute setting the
uric acid stones are unlikely to be coated with calcium and will therefore be
radiolucent.
Next question
Renal stones
Type of Percentage of
stones Features all calculi
Type of Percentage of
stones Features all calculi
oxalate causes)
Hyperoxaluria may also increase risk
Hypocitraturia increases risk because citrate
forms complexes with calcium making it more
soluble
Stones are radio-opaque (though less than
calcium phosphate stones)
Hyperuricosuria may cause uric acid stones to
which calcium oxalate binds
Next question
Save my notes
Question stats
A 8%
B 8.3%
C 15.2%
D 8.1%
E 60.4%
Search eMRCS
Question 32 of 63
gathered by dr. elbarky, for free, not intended for profit by anybody elsewhere
From the list below, which drug is known to cause haemorrhagic cystitis?
Rifampicin
Methotrexate
Dexamethasone
Leflunomide
Cyclophosphamide
Next question
Haematuria
Causes of haematuria
Infection Remember TB
Malignancy Renal cell carcinoma (remember paraneoplastic
syndromes): painful or painless
Urothelial malignancies: 90% are transitional cell
gathered by dr. elbarky, for free, not intended for profit by anybody elsewhere
Benign Exercise
Iatrogenic Catheterisation
Radiotherapy; cystitis, severe haemorrhage,
bladder necrosis
References
Http://bestpractice.bmj.com/best-practice/monograph/316/overview
/aetiology.html
Next question
gathered by dr. elbarky, for free, not intended for profit by anybody elsewhere
Save my notes
Question stats
A 11.6%
B 14.3%
C 6.2%
D 9.1%
E 58.8%
Search eMRCS
Search term Go
External links
+ Suggest a link
Dashboard
3
Question 33 of 63
gathered by dr. elbarky, for free, not intended for profit by anybody elsewhere
Transvesical prostatectomy
Radical prostatectomy
Chemotherapy alone
Chemical orchidectomy
Next question
Prostate Cancer
Prostate Cancer
This is a common condition and up to 30,000 men are diagnosed with the
condition each year. Up to 9,000 will die in in the UK from the condition per year.
Diagnosis
Early prostate cancers have few symptoms.
Metastatic disease may present as bone pain.
Locally advanced disease may present as pelvic pain or with urinary symptoms.
Prostate specific antigen measurement
Digital rectal examination
Trans rectal USS (+/- biopsy)
MRI/ CT and bone scan for staging.
PSA Test
The normal upper limit for PSA is 4ng/ml. However, in this group will lie patients
gathered by dr. elbarky, for free, not intended for profit by anybody elsewhere
with benign disease and some with localised prostate cancer. False positives may
be due to prostatitis, UTI, BPH, vigorous DRE.
The percentage of free: total PSA may help to distinguish benign disease from
cancer. Values of <20% are suggestive of cancer and biopsy is advised.
Pathology
95% adenocarcinoma
In situ malignancy is sometimes found in areas adjacent to cancer. Multiple
biopsies needed to call true in situ disease.
Often multifocal- 70% lie in the peripheral zone.
Graded using the Gleason grading system, two grades awarded 1 for most
dominant grade (on scale of 1-5) and 2 for second most dominant grade
(scale 1-5). The two added together give the Gleason score. Where 2 is best
prognosis and 10 the worst.
Lymphatic spread occurs first to the obturator nodes and local extra
prostatic spread to the seminal vesicles is associated with distant disease.
Treatment
Watch and wait- Elderly, multiple co-morbidities, low Gleason score
Radiotherapy (External)- Both potentially curative and palliative therapy
possible. However, radiation proctitis and rectal malignancy are late
problems. Brachytherapy is a modification allowing internal radiotherapy.
Surgery- Radical prostatectomy. Surgical removal of the prostate is the
standard treatment for localised disease. The robot is being used
increasingly for this procedure. As well as the prostate the obturator nodes
are also removed to complement the staging process. Erectile dysfunction
is a common side effect. Survival may be better than with radiotherapy (see
references).
Hormonal therapy- Testosterone stimulates prostate tissue and prostatic
cancers usually show some degree of testosterone dependence. 95% of
testosterone is derived from the testis and bilateral orchidectomy may be
used for this reason. Pharmacological alternatives include LHRH analogues
and anti androgens (which may be given in combination).
In the UK the National Institute for Clinical Excellence (NICE) suggests that
active surveillance is the preferred option for low risk men. It is particularly
suitable for men with clinical stage T1c, Gleason score 3+3 and PSA density
< 0.15 ng/ml/ml who have cancer in less than 50% of their biopsy cores,
with < 10 mm of any core involved.
References
1. Prostate cancer pathway. NICE.(http://guidance.nice.org.uk/IPG424)
gathered by dr. elbarky, for free, not intended for profit by anybody elsewhere
Next question
Save my notes
Question stats
A 12.9%
B 28.6%
C 40.4%
D 7.5%
E 10.5%
Search eMRCS
Search term Go
External links
Question 34 of 63
gathered by dr. elbarky, for free, not intended for profit by anybody elsewhere
Rifampicin
Phosphaturia
Beetroot
Rhubarb
Blackberries
Next question
Haematuria
Causes of haematuria
Infection Remember TB
Malignancy Renal cell carcinoma (remember paraneoplastic
syndromes): painful or painless
Urothelial malignancies: 90% are transitional cell
gathered by dr. elbarky, for free, not intended for profit by anybody elsewhere
Benign Exercise
Iatrogenic Catheterisation
Radiotherapy; cystitis, severe haemorrhage,
bladder necrosis
References
Http://bestpractice.bmj.com/best-practice/monograph/316/overview
/aetiology.html
Next question
gathered by dr. elbarky, for free, not intended for profit by anybody elsewhere
Save my notes
Question stats
A 8.1%
B 46.4%
C 6.7%
D 13.9%
E 24.9%
Search eMRCS
Search term Go
External links
+ Suggest a link
Dashboard
1
Question 35 of 63
gathered by dr. elbarky, for free, not intended for profit by anybody elsewhere
Cystogram
Suspected penile fractures should be surgically explored and the injury repaired.
Next question
Penile fracture
Penile fractures are a rare type of urological trauma that may be encountered. The
injury is usually in the proximal part of the penile shaft and may involve the urethra.
A classically history of a snapping sensation followed by immediate pain is usually
given by the patient (usually during vigorous intercourse). On examination there is
usually a tense haematoma and blood may be seen at the meatus if the urethra is
injured.
When there is a a strong suspicion of the diagnosis the correct management is
surgical and a circumferential incision made immediately inferior to the glans. The
skin and superficial tissues are stripped back and the penile shaft inspected.
Injuries are usually sutured and the urethra repaired over a catheter.
Next question
gathered by dr. elbarky, for free, not intended for profit by anybody elsewhere
Save my notes
Question stats
A 12.2%
B 56.9%
C 7%
D 15.2%
E 8.7%
Search eMRCS
Search term Go
External links
+ Suggest a link
Dashboard
3
Question 36 of 63
gathered by dr. elbarky, for free, not intended for profit by anybody elsewhere
Neurofibromatosis
Budd-Chiari syndrome
Tuberous sclerosis
Next question
Tuberous sclerosis
Cutaneous features
depigmented 'ash-leaf' spots which fluoresce under UV light
roughened patches of skin over lumbar spine (Shagreen patches)
adenoma sebaceum: butterfly distribution over nose
fibromata beneath nails (subungual fibromata)
café-au-lait spots* may be seen
Neurological features
developmental delay
epilepsy (infantile spasms or partial)
intellectual impairment
gathered by dr. elbarky, for free, not intended for profit by anybody elsewhere
Also
retinal hamartomas: dense white areas on retina (phakomata)
rhabdomyomas of the heart
gliomatous changes can occur in the brain lesions
polycystic kidneys, renal angiomyolipomata
Next question
Save my notes
Question stats
A 9.9%
B 8.9%
C 16%
D 28.5%
E 36.7%
Search eMRCS
Search term Go
Question 37 of 63
gathered by dr. elbarky, for free, not intended for profit by anybody elsewhere
A 43 year old lady with episodes of recurrent urinary tract sepsis presents with a
staghorn calculus of the left kidney. Her urinary pH is 7.8. A KUB x-ray shows a
faint outline of the calculus. What is the most likely stone composition?
Struvite
Calcium phosphate
Calcium oxalate
Uric acid
Cystine
Chronic infection with urease producing enzymes can produce an alkaline urine
with formation of struvite stone.
Next question
Renal stones
Type of Percentage of
stones Features all calculi
Type of Percentage of
stones Features all calculi
oxalate causes)
Hyperoxaluria may also increase risk
Hypocitraturia increases risk because citrate
forms complexes with calcium making it more
soluble
Stones are radio-opaque (though less than
calcium phosphate stones)
Hyperuricosuria may cause uric acid stones to
which calcium oxalate binds
Next question
Save my notes
Question stats
A 65.7%
B 8%
C 9.3%
D 8.5%
E 8.5%
Search eMRCS
Question 38 of 63
gathered by dr. elbarky, for free, not intended for profit by anybody elsewhere
Sacral neuromodulation
Bladder drill
Oxybutinin
Colposuspension
Overactive bladder syndrome is very common and first line management includes
the use of anticholinergics and bladder drill whereby voiding is deferred. Refractory
cases can be treated with SNS or botulinum toxin injections. A Burch
Colposuspension is used to treat stress urinary incontinence.
Next question
Urinary incontinence
Involuntary passage of urine. Most cases are female (80%). It has a prevalence of
11% in those aged greater than 65 years. The commonest variants include:
Stress urinary incontinence (50%)
Urge incontinence (15%)
Mixed (35%)
Males
Males may also suffer from incontinence although it is a much rarer condition in
men. A number of anatomical factors contribute to this. Males have 2 powerful
sphincters; one at the bladder neck and the other in the urethra. Damage to the
bladder neck mechanism is a factor in causing retrograde ejaculation following
prostatectomy. The short segment of urethra passing through the urogenital
diaphragm consists of striated muscle fibres (the external urethral sphincter) and
smooth muscle capable of more sustained contraction. It is the latter mechanism
gathered by dr. elbarky, for free, not intended for profit by anybody elsewhere
Females
The sphincter complex at the level of bladder neck is poorly developed in females.
As a result the external sphincter complex is functionally more important, its
composition being similar to that of males. Innervation is via the pudendal nerve
and the neuropathy that may accompany obstetric events may compromise this
and lead to stress urinary incontinence.
Innervation
Somatic innervation to the bladder is via the pudendal, hypogastric and pelvic
nerves. Autonomic nerves travel in these nerve fibres too. Bladder filling leads to
detrusor relaxation (sympathetic) coupled with sphincter contraction. The
parasympathetic system causes detrusor contraction and sphincter relaxation.
Overall control of micturition is centrally mediated via centres in the Pons.
Urethral mobility:
Pressure not transmitted appropriately to the urethra resulting in involuntary
passage of urine during episodes of raised intra-abdominal pressure.
Sphincter dysfunction:
Sphincter fails to adapt to compress urethra resulting in involuntary passage of
urine. When the sphincter completely fails there is often to continuous passage of
urine.
Urge incontinence
In these patients there is sense of urgency followed by incontinence. The detrusor
muscle in these patients is unstable and urodynamic investigation will
demonstrate overactivity of the detrusor muscle at inappropriate times (e.g.
Bladder filling). Urgency may be seen in patients with overt neurological disorders
and those without. The pathophysiology is not well understood but poor central
and peripheral co-ordination of the events surrounding bladder filling are the main
processes.
Assessment
Careful history and examination including vaginal examination for cystocele.
Bladder diary for at least 3 days
Consider flow cystometry if unclear symptomatology or surgery considered and
diagnosis is unclear.
Exclusion of other organic disease (e.g. Stones, UTI, Cancer)
gathered by dr. elbarky, for free, not intended for profit by anybody elsewhere
Management
Conservative measures should be tried first; Stress urinary incontinence or mixed
symptoms should undergo 3 months of pelvic floor exercise. Over active bladder
should have 6 weeks of bladder retraining.
Drug therapy for women with overactive bladder should be offered oxybutynin (or
solifenacin if elderly) if conservative measures fail.
In women with detrusor instability who fail non operative therapy a trial of sacral
neuromodulation may be considered, with conversion to permanent implant if
good response. Augmentation cystoplasty is an alternative but will involve long
term intermittent self catheterisation.
In women with stress urinary incontinence a urethral sling type procedure may be
undertaken. Where cystocele is present in association with incontinence it should
be repaired particularly if it lies at the introitus.
NICE guidelines
Initial assessment urinary incontinence should be classified as stress/urge
/mixed.
At least 3/7 bladder diary if unable to classify easily.
Start conservative treatment before urodynamic studies if a diagnosis is
obvious from the history
Urodynamic studies if plans for surgery.
Stress incontinence: Pelvic floor exercises 3/12, if fails consider surgery.
Urge incontinence: Bladder training >6/52, if fails for oxybutynin
(antimuscarinic drugs) then sacral nerve stimulation.
Pelvic floor exercises offered to all women in their 1st pregnancy.
Next question
Save my notes
Question stats
A 8%
B 13.5%
C 16.6%
D 10.4%
E 51.5%
gathered by dr. elbarky, for free, not intended for profit by anybody elsewhere
Search eMRCS
Search term Go
External links
+ Suggest a link
Dashboard
10
11
12
13
14
Question 39 of 63
gathered by dr. elbarky, for free, not intended for profit by anybody elsewhere
A 65 year old man presents with significant lower urinary tract symptoms and is
diagnosed as having benign prostatic hyperplasia. Which of the following drug
treatments will produce the slowest clinical response?
Tamsulosin
Alfuzosin
Doxazosin
Finasteride
Terazosin
5 alpha reductase inhibitors have a more favorable side effect profile than α
blockers.
Alpha blockers have a faster onset of action (but lower reduction of complications
from BPH) than 5 α reductase inhibitors.
Next question
Benign prostatic hyperplasia occurs via an increase in the epithelial and stromal
cell numbers in the peri-urethral zone of the prostate. BPH is very common and
90% of men aged over 80 will have at least microscopic evidence of benign
prostatic hyperplasia. The causes of BPH are still not well understood, but the
importance of androgens remains appreciated even if the exact role by which they
induce BPH is elusive.
Presentation
The vast majority of men will present with lower urinary tract symptoms. These will
typically be:
gathered by dr. elbarky, for free, not intended for profit by anybody elsewhere
Poor flow
Nocturia
Hesitancy
Incomplete and double voiding
Terminal dribbling
Urgency
Incontinence
Investigation
Digital rectal examination to assess prostatic size and morphology.
Urine dipstick for infections and haematuria.
Uroflowmetry (a flow rate of >15ml/second helps to exclude BOO)
Bladder pressure studies may help identify detrusor failure and whilst may
not form part of first line investigations should be included in those with
atypical symptoms and prior to redo surgery.
Bladder scanning to demonstrate residual volumes. USS if high pressure
chronic retention.
Management
Lifestyle changes such as stopping smoking and altering fluid intake may
help those with mild symptoms.
Medical therapy includes alpha blockers and 5 α reductase inhibitors. The
former work quickly on receptor zones located at the bladder neck.
Cardiovascular side effects are well documented. The latter work on
testosterone metabolising enzymes. Although they have a slower onset of
action, the 5 α reductase inhibitors may prevent acute urinary retention.
Surgical therapy includes transurethral resection of the prostate and is the
treatment of choice in those with severe symptoms and those who fail to
respond to medical therapy. More tailored bladder neck incision procedures
may be considered in those with small prostates. Retrograde ejaculation
may occur following surgery. The change in the type of irrigation solutions
used has helped to minimise the TURP syndrome of electrolyte
disturbances.
Next question
Save my notes
gathered by dr. elbarky, for free, not intended for profit by anybody elsewhere
Question stats
A 13.8%
B 9%
C 11.1%
D 56.6%
E 9.4%
Search eMRCS
Search term Go
External links
+ Suggest a link
Dashboard
8
Question 40 of 63
gathered by dr. elbarky, for free, not intended for profit by anybody elsewhere
A parent brings her 4 year old child to the surgical clinic. She has noticed an
intermittent swelling in the right scrotum that is worse in the evening. On
examination he has a soft fluctuant swelling in the right scrotum that cannot be
separated from the testis. It transilluminates when a pen torch is held against it.
What is the best course of action?
Aspiration
Next question
Scrotal swelling
Differential diagnosis
Management
Testicular malignancy is always treated with orchidectomy via an inguinal
approach. This allows high ligation of the testicular vessels and avoids
exposure of another lymphatic field to the tumour.
Torsion is commonest in young teenagers and the history in older children
can be difficult to elicit. Intermittent torsion is a recognised problem. The
treatment is prompt surgical exploration and testicular fixation. This can be
achieved using sutures or by placement of the testis in a Dartos pouch.
Varicoceles are usually managed conservatively. If there are concerns about
testicular function of infertility then surgery or radiological management can
gathered by dr. elbarky, for free, not intended for profit by anybody elsewhere
be considered.
Epididymal cysts can be excised using a scrotal approach
Hydroceles are managed differently in children where the underlying
pathology is a patent processus vaginalis and therefore an inguinal
approach is used in children so that the processus can be ligated. In adults
a scrotal approach is preferred and the hydrocele sac excised or plicated.
Next question
Save my notes
Question stats
A 58.7%
B 15%
C 12.8%
D 7.9%
E 5.6%
Search eMRCS
Search term Go
Question 41 of 63
gathered by dr. elbarky, for free, not intended for profit by anybody elsewhere
A 55 year old man has multiple stones within the renal pelvis, including one
measuring 25mm in diameter. What is the most appropriate course of action?
Arrange a nephrectomy
Conservative management
The intervention of choice for renal pelvis stones greater than 20mm is PCNL. This
stone is highly unlikely to pass spontaneously and ureteroscopy for a stone of this
size would be extremely time-consuming.
Next question
Management
Most renal stones measuring less than 5mm in maximum diameter will typically
pass within 4 weeks of symptom onset(1)(2). More urgent treatment is indicated in
the presence of ureteric obstruction, renal developmental abnormalities such as
horseshoe kidney and previous renal transplant. Ureteric obstruction together with
infection is a surgical emergency and the system must be decompressed. Options
include nephrostomy tube placement or ureteric stent placement via cystoscopy.
In the non-emergency setting the preferred options for treatment of stone disease
gathered by dr. elbarky, for free, not intended for profit by anybody elsewhere
Ureteroscopy
A ureteroscope is passed retrograde via the urethra and bladder, into the ureter and
renal pelvis. Laser or pneumatic fragmentation (lithoclasty) is performed using the
ureteroscope and stone fragments are extracted(3). In some cases a stent is left in
situ after the procedure.
Percutaneous nephrolithotomy
In this procedure percutaneous access is gained to the renal collecting system.
Once access is achieved, endoscopic intra-corporeal lithotripsy, lithoclasty or laser
stone fragmentation is performed and stone fragments removed.
Therapeutic selection
Renal stones
Ureteric stones
5-10mm ESWL
gathered by dr. elbarky, for free, not intended for profit by anybody elsewhere
10-20mm Ureteroscopy
Reference
1. Bultitude M, Rees J. Management of renal colic. BMJ. 2012;345(7872):18.
2. Shah TT, Gao C, Peters M, Manning T, Cashman S, Nambiar A, et al. Factors
associated with spontaneous stone passage in a contemporary cohort of patients
presenting with acute ureteric colic: results from the Multi-centre cohort study
evaluating the role of Inflammatory Markers In patients presenting with acute
ureteric . BJU Int. 2019 Apr.
3. BAUS. Ureteroscopy (Telescopic surgery for stone removal) (Internet). BAUS
patient information leaflet. 2017 (cited 2019 Jun 27). p. 16. Available from:
https://www.baus.org.uk/userfiles/pages/files/Patients/Leaflets/Ureteroscopy for
stone.pdf
4. BAUS. Percutaneous nephrolithotomy (keyhole surgery for kidney stones). BAUS
patient information leaflet. 2017. p. 16. Available from: https://www.baus.org.uk
/userfiles/pages/files/Patients/Leaflets/PCNL.pdf
5. NICE. Renal and ureteric stones: assessment and management. NICE
Guide[NG118]. 2019;(January):112.
Next question
Save my notes
Question stats
A 6.7%
B 50.7%
C 19.7%
D 10.8%
E 12.1%
Question 42 of 63
gathered by dr. elbarky, for free, not intended for profit by anybody elsewhere
A 16 year old boy presents with renal colic. His parents both have a similar history
of the condition. His urine tests positive for blood. A KUB style x-ray shows a
relatively radiodense stone in the region of the mid ureter. What is the most likely
composition of the stone?
Struvite stone
Cystine stone
Next question
Renal stones
Type of Percentage of
stones Features all calculi
Type of Percentage of
stones Features all calculi
oxalate causes)
Hyperoxaluria may also increase risk
Hypocitraturia increases risk because citrate
forms complexes with calcium making it more
soluble
Stones are radio-opaque (though less than
calcium phosphate stones)
Hyperuricosuria may cause uric acid stones to
which calcium oxalate binds
Next question
Save my notes
Question stats
A 19.1%
B 8.8%
C 9.2%
D 43.2%
E 19.7%
Search eMRCS
Question 43 of 63
gathered by dr. elbarky, for free, not intended for profit by anybody elsewhere
A 68 year old man has a TCC of the bladder. He has a right hydronephrosis
detected on ultrasound and deteriorating renal function. A DMSA scan shows a
non functioning left kidney. At cystoscopy the ureteric orifice cannot be readily
accessed. What is the best course of action?
Radiotherapy
A TCC occluding the ureteric orifice will obscure its identification during surgery,
so that passage of a retrograde stent is difficult. Therefore passage of a stent
from the renal pelvis is preferable.
Next question
Hydronephrosis
Causes of hydronephrosis
Unilateral: PACT
Pelvic-ureteric obstruction (congenital or acquired)
Aberrant renal vessels
Calculi
Tumours of renal pelvis
gathered by dr. elbarky, for free, not intended for profit by anybody elsewhere
Bilateral: SUPER
Stenosis of the urethra
Urethral valve
Prostatic enlargement
Extensive bladder tumour
Retro-peritoneal fibrosis
Investigation
USS- identifies presence of hydronephrosis and can assess the kidneys
IVU- assess the position of the obstruction
Antegrade or retrograde pyelography- allows treatment
If renal colic suspected: non contrast CT scan (majority of stones are
detected this way)
Management
Remove the obstruction and drainage of urine
Acute upper urinary tract obstruction: Nephrostomy tube
Chronic upper urinary tract obstruction: Ureteric stent or a pyeloplasty
Next question
Save my notes
Question stats
A 45.5%
B 22.5%
gathered by dr. elbarky, for free, not intended for profit by anybody elsewhere
C 18%
D 5.3%
E 8.7%
Search eMRCS
Search term Go
External links
+ Suggest a link
Dashboard
10
11
12
Question 44 of 63
gathered by dr. elbarky, for free, not intended for profit by anybody elsewhere
A 24 year old man presents with a persistent and unwanted erection that has been
present for the previous 6 hours. On examination the penis is rigid and tender.
Aspiration of blood from the corpus cavernosa shows dark blood. Which of the
following is the most appropriate initial management?
Low flow priapism is a urological emergency. Aspiration of bright red blood is more
reassuring and may indicate high flow priapism that may be actively monitored.
Low flow priapism should be decompressed with aspiration of blood from the
corpus cavernosum.
Next question
Penile erection
Physiology of erection
Autonomic discharge to the penis will trigger the veno-occlusive mechanism which
triggers the flow of arterial blood into the penile sinusoidal spaces. As the inflow
increases the increased volume in this space will secondarily lead to compression
of the subtunical venous plexus with reduced venous return. During the
detumesence phase the arteriolar constriction will reduce arterial inflow and
thereby allow venous return to normalise.
Priapism
Prolonged unwanted erection, in the absence of sexual desire, lasting more than 4
hours.
Classification of priapism
Causes
Intracavernosal drug therapies (e.g. for erectile dysfunction>
Blood disorders such as leukaemia and sickle cell disease
Neurogenic disorders such as spinal cord transection
Trauma to penis resulting in arterio-venous malformations
Tests
Exclude sickle cell/ leukaemia
Consider blood sampling from cavernosa to determine whether high or low
flow (low flow is often hypoxic)
Management
Ice packs/ cold showers
If due to low flow then blood may be aspirated from copora or try
intracavernosal alpha adrenergic agonists.
Delayed therapy of low flow priapism may result in erectile dysfunction.
Next question
gathered by dr. elbarky, for free, not intended for profit by anybody elsewhere
Save my notes
Question stats
A 5.8%
B 10.4%
C 58%
D 11.5%
E 14.2%
Search eMRCS
Search term Go
External links
+ Suggest a link
Dashboard
1
Question 45 of 63
gathered by dr. elbarky, for free, not intended for profit by anybody elsewhere
A 43 year old female has undergone a renal transplant 12 months previously. Over
the past few weeks there have been concerns about deteriorating renal function.
What is the most useful investigation?
MAG 3 renogram
DMSA scan
Renal CT scan
Micturating cystourethrohram
Next question
DMSA scan
Dimercaptosuccinic acid (DMSA) scintigraphy
DMSA localises to the renal cortex with little accumulation in the renal papilla and
medulla. It is useful for the identification of cortical defects and ectopic or
aberrant kidneys. It does not provide useful information on the ureter of collecting
system.
agent of choice for imaging the kidneys of patients with existing renal impairment
(where GFR is impaired).
PET/CT
This may be used to evaluate structurally indeterminate lesions in the staging of
malignancy.
References
Davis A et al. Investigating urinary tract infections in children. BMJ 2013
(346):35-37.
Next question
Save my notes
Question stats
A 53%
B 26.1%
C 7.1%
Question 46 of 63
gathered by dr. elbarky, for free, not intended for profit by anybody elsewhere
Schistosoma mansoni
Sarcoidosis
Leishmaniasis
Tuberculosis
Schistosoma haematobium
Next question
Schistosomiasis
Schistosoma haematobium
This typically presents as a 'swimmer's itch' in patients who have recently returned
from Africa. Schistosoma haematobium is a risk factor for squamous cell bladder
cancer
gathered by dr. elbarky, for free, not intended for profit by anybody elsewhere
Features
Frequency
Haematuria
Bladder calcification
Management
Single oral dose of praziquantel
Next question
Save my notes
Question stats
A 26.4%
B 7%
C 10.3%
D 8.2%
E 48.1%
Search eMRCS
Search term Go
Question 47 of 63
gathered by dr. elbarky, for free, not intended for profit by anybody elsewhere
A 13 month old boy is brought to the paediatric clinic by his mother who is
concerned that his testis are not palpable. On examination his testis are not
palpable either in the scrotum or inguinal region and cannot be visualised on
ultrasound either. What is the most appropriate next stage in management?
Laparoscopy
Administration of testosterone
Next question
Cryptorchidism
A congenital undescended testis is one that has failed to reach the bottom of the
scrotum by 3 months of age. At birth up to 5% of boys will have an undescended
testis, post natal descent occurs in most and by 3 months the incidence of
cryptorchidism falls to 1-2%. In the vast majority of cases the cause of the
maldescent is unknown. A proportion may be associated with other congenital
defects including:
Differential diagnosis
These include retractile testes and, in the case of absent bilateral testes the
possibility of intersex conditions. A retractile testis can be brought into the
scrotum by the clinician and when released remains in the scrotum. If the
examining clinician notes the testis to return rapidly into the inguinal canal when
released then surgery is probably indicated.
Males with undescended testis are 40 times as likely to develop testicular cancer
(seminoma) as males without undescended testis
The location of the undescended testis affects the relative risk of testicular cancer
(50% intra-abdominal testes)
Treatment
Orchidopexy at 6- 18 months of age. The operation usually consists of
inguinal exploration, mobilisation of the testis and implantation into a dartos
pouch.
Intra-abdominal testis should be evaluated laparoscopically and mobilised.
Whether this is a single stage or two stage procedure depends upon the
exact location.
After the age of 2 years in untreated individuals the Sertoli cells will degrade
and those presenting late in teenage years may be better served by
orchidectomy than to try and salvage a non functioning testis with an
increased risk of malignancy.
Next question
Save my notes
Question stats
A 57.8%
gathered by dr. elbarky, for free, not intended for profit by anybody elsewhere
B 19.3%
C 6.6%
D 8.3%
E 7.9%
Search eMRCS
Search term Go
External links
+ Suggest a link
Dashboard
10
Question 48 of 63
gathered by dr. elbarky, for free, not intended for profit by anybody elsewhere
EuroQOL score of 5
Gleason score of 2
Gleason score of 10
Prostate cancer is histologically graded using the Gleason score (see below). A
score of 10 is consistent with a histologically aggressive form of the disease. The
FIGO staging system is used to stage gynaecological malignancy. The EuroQOL
score is a quality of life measurement tool.
Next question
Prostate Cancer
Prostate Cancer
This is a common condition and up to 30,000 men are diagnosed with the
condition each year. Up to 9,000 will die in in the UK from the condition per year.
Diagnosis
Early prostate cancers have few symptoms.
Metastatic disease may present as bone pain.
Locally advanced disease may present as pelvic pain or with urinary symptoms.
Prostate specific antigen measurement
Digital rectal examination
Trans rectal USS (+/- biopsy)
MRI/ CT and bone scan for staging.
PSA Test
The normal upper limit for PSA is 4ng/ml. However, in this group will lie patients
gathered by dr. elbarky, for free, not intended for profit by anybody elsewhere
with benign disease and some with localised prostate cancer. False positives may
be due to prostatitis, UTI, BPH, vigorous DRE.
The percentage of free: total PSA may help to distinguish benign disease from
cancer. Values of <20% are suggestive of cancer and biopsy is advised.
Pathology
95% adenocarcinoma
In situ malignancy is sometimes found in areas adjacent to cancer. Multiple
biopsies needed to call true in situ disease.
Often multifocal- 70% lie in the peripheral zone.
Graded using the Gleason grading system, two grades awarded 1 for most
dominant grade (on scale of 1-5) and 2 for second most dominant grade
(scale 1-5). The two added together give the Gleason score. Where 2 is best
prognosis and 10 the worst.
Lymphatic spread occurs first to the obturator nodes and local extra
prostatic spread to the seminal vesicles is associated with distant disease.
Treatment
Watch and wait- Elderly, multiple co-morbidities, low Gleason score
Radiotherapy (External)- Both potentially curative and palliative therapy
possible. However, radiation proctitis and rectal malignancy are late
problems. Brachytherapy is a modification allowing internal radiotherapy.
Surgery- Radical prostatectomy. Surgical removal of the prostate is the
standard treatment for localised disease. The robot is being used
increasingly for this procedure. As well as the prostate the obturator nodes
are also removed to complement the staging process. Erectile dysfunction
is a common side effect. Survival may be better than with radiotherapy (see
references).
Hormonal therapy- Testosterone stimulates prostate tissue and prostatic
cancers usually show some degree of testosterone dependence. 95% of
testosterone is derived from the testis and bilateral orchidectomy may be
used for this reason. Pharmacological alternatives include LHRH analogues
and anti androgens (which may be given in combination).
In the UK the National Institute for Clinical Excellence (NICE) suggests that
active surveillance is the preferred option for low risk men. It is particularly
suitable for men with clinical stage T1c, Gleason score 3+3 and PSA density
< 0.15 ng/ml/ml who have cancer in less than 50% of their biopsy cores,
with < 10 mm of any core involved.
References
1. Prostate cancer pathway. NICE.(http://guidance.nice.org.uk/IPG424)
gathered by dr. elbarky, for free, not intended for profit by anybody elsewhere
Next question
Save my notes
Question stats
A 6.4%
B 12.6%
C 6.5%
D 9%
E 65.5%
Search eMRCS
Search term Go
External links
Question 49 of 63
gathered by dr. elbarky, for free, not intended for profit by anybody elsewhere
A 44 year old man is referred to the clinic because of a swelling and discomfort in
the right scrotum. This is present most of the time and he is otherwise well with no
urinary symptoms. On examination, he has a soft, fluctuant swelling in the right
scrotum that transilluminates easily. An ultrasound is performed that confirms that
the underlying testicle is structurally normal. What is the best course of action?
Adult hydroceles are less commonly due to the persistence of embryonic remnants
and therefore can be managed via a scrotal approach. Both the Lords and
Jaboulay procedures are reasonable options. However, only a scrotal approach
should be adopted.
Next question
Scrotal swelling
Differential diagnosis
Management
Testicular malignancy is always treated with orchidectomy via an inguinal
approach. This allows high ligation of the testicular vessels and avoids
exposure of another lymphatic field to the tumour.
Torsion is commonest in young teenagers and the history in older children
can be difficult to elicit. Intermittent torsion is a recognised problem. The
treatment is prompt surgical exploration and testicular fixation. This can be
achieved using sutures or by placement of the testis in a Dartos pouch.
Varicoceles are usually managed conservatively. If there are concerns about
testicular function of infertility then surgery or radiological management can
gathered by dr. elbarky, for free, not intended for profit by anybody elsewhere
be considered.
Epididymal cysts can be excised using a scrotal approach
Hydroceles are managed differently in children where the underlying
pathology is a patent processus vaginalis and therefore an inguinal
approach is used in children so that the processus can be ligated. In adults
a scrotal approach is preferred and the hydrocele sac excised or plicated.
Next question
Save my notes
Question stats
A 15.8%
B 12.3%
C 47.8%
D 16.3%
E 7.8%
Search eMRCS
Search term Go
Question 50 of 63
gathered by dr. elbarky, for free, not intended for profit by anybody elsewhere
A 33 year old man presents with a painless lump in his left testis. USS and blood
tests are suspicious for teratoma. What is the most appropriate next step?
Next question
Testicular disorders
Testicular cancer
Testicular cancer is the most common malignancy in men aged 20-30 years.
Around 95% of cases of testicular cancer are germ-cell tumours. Germ cell
tumours may essentially be divided into:
Tumour
Tumour type Key features markers Pathology
Tumour
Tumour type Key features markers Pathology
(https://d2zgo9qer4wjf4.cloudfront.net/images_eMRCS/swb090b.jpg)
Image sourced from Wikipedia
(https://d2zgo9qer4wjf4.cloudfront.net
(http://en.wikipedia.org
/images_eMRCS/swb090b.jpg)
/wiki/Seminoma)
Features
A painless lump is the most common presenting symptom
Pain may also be present in a minority of men
Other possible features include hydrocele, gynaecomastia
Diagnosis
Ultrasound is first-line
CT scanning of the chest/ abdomen and pelvis is used for staging
Tumour markers (see above) should be measured
gathered by dr. elbarky, for free, not intended for profit by anybody elsewhere
Management
Orchidectomy (Inguinal approach)
Chemotherapy and radiotherapy may be given depending on staging
Abdominal lesions >1cm following chemotherapy may require
retroperitoneal lymph node dissection.
Benign disease
Epididymo-orchitis
Acute epididymitis is an acute inflammation of the epididymis, often involving the
testis and usually caused by bacterial infection.
Infection spreads from the urethra or bladder. In men <35 years, gonorrhoea
or chlamydia are the usual infections.
Amiodarone is a recognised non infective cause of epididymitis, which
resolves on stopping the drug.
Tenderness is usually confined to the epididymis, which may facilitate
differentiating it from torsion where pain usually affects the entire testis.
Testicular torsion
Twist of the spermatic cord resulting in testicular ischaemia and necrosis.
Most common in males aged between 10 and 30 (peak incidence 13-15
years)
Pain is usually severe and of sudden onset.
Cremasteric reflex is lost and elevation of the testis does not ease the pain.
Treatment is with surgical exploration. If a torted testis is identified then
both testis should be fixed as the condition of bell clapper testis is often
bilateral.
Hydrocele
Presents as a mass that transilluminates, usually possible to 'get above' it
on examination.
In younger men it should be investigated with USS to exclude tumour.
In children it may occur as a result of a patent processus vaginalis.
Treatment in adults is with a Lords or Jabouley procedure.
Treatment in children is with trans inguinal ligation of PPV.
Next question
gathered by dr. elbarky, for free, not intended for profit by anybody elsewhere
Save my notes
Question stats
A 60.9%
B 13.7%
C 10.8%
D 8.8%
E 5.8%
Search eMRCS
Search term Go
External links
+ Suggest a link
Dashboard
3
Question 51 of 63
gathered by dr. elbarky, for free, not intended for profit by anybody elsewhere
A 75 year old man presents with locally advanced carcinoma of the prostate and
vertebral body metastasis with impending spinal cord compression. Which of the
following agents (if used in isolation) carries the greatest risk of worsening his
symptoms in the short term?
Surgical orchidectomy
Cyproterone acetate
Flutamide
LHRH analogues may cause flare of metastatic disease and anti androgens should
be administered to counter this. Surgical orchidectomy reduces testosterone levels
within 8 hours (but fails to reduce adrenal androgen release). Cyproterone and
flutamide are androgen blockers that may be considered as add on therapy to
reduce the risk of tumour flare when commencing treatment with LH RH
analogues.
Next question
Prostate Cancer
Prostate Cancer
This is a common condition and up to 30,000 men are diagnosed with the
condition each year. Up to 9,000 will die in in the UK from the condition per year.
Diagnosis
Early prostate cancers have few symptoms.
Metastatic disease may present as bone pain.
Locally advanced disease may present as pelvic pain or with urinary symptoms.
Prostate specific antigen measurement
Digital rectal examination
Trans rectal USS (+/- biopsy)
gathered by dr. elbarky, for free, not intended for profit by anybody elsewhere
PSA Test
The normal upper limit for PSA is 4ng/ml. However, in this group will lie patients
with benign disease and some with localised prostate cancer. False positives may
be due to prostatitis, UTI, BPH, vigorous DRE.
The percentage of free: total PSA may help to distinguish benign disease from
cancer. Values of <20% are suggestive of cancer and biopsy is advised.
Pathology
95% adenocarcinoma
In situ malignancy is sometimes found in areas adjacent to cancer. Multiple
biopsies needed to call true in situ disease.
Often multifocal- 70% lie in the peripheral zone.
Graded using the Gleason grading system, two grades awarded 1 for most
dominant grade (on scale of 1-5) and 2 for second most dominant grade
(scale 1-5). The two added together give the Gleason score. Where 2 is best
prognosis and 10 the worst.
Lymphatic spread occurs first to the obturator nodes and local extra
prostatic spread to the seminal vesicles is associated with distant disease.
Treatment
Watch and wait- Elderly, multiple co-morbidities, low Gleason score
Radiotherapy (External)- Both potentially curative and palliative therapy
possible. However, radiation proctitis and rectal malignancy are late
problems. Brachytherapy is a modification allowing internal radiotherapy.
Surgery- Radical prostatectomy. Surgical removal of the prostate is the
standard treatment for localised disease. The robot is being used
increasingly for this procedure. As well as the prostate the obturator nodes
are also removed to complement the staging process. Erectile dysfunction
is a common side effect. Survival may be better than with radiotherapy (see
references).
Hormonal therapy- Testosterone stimulates prostate tissue and prostatic
cancers usually show some degree of testosterone dependence. 95% of
testosterone is derived from the testis and bilateral orchidectomy may be
used for this reason. Pharmacological alternatives include LHRH analogues
and anti androgens (which may be given in combination).
In the UK the National Institute for Clinical Excellence (NICE) suggests that
active surveillance is the preferred option for low risk men. It is particularly
suitable for men with clinical stage T1c, Gleason score 3+3 and PSA density
< 0.15 ng/ml/ml who have cancer in less than 50% of their biopsy cores,
with < 10 mm of any core involved.
treatment. Treatment decisions should be made with the man, taking into account
co-morbidities and life expectancy.
References
1. Prostate cancer pathway. NICE.(http://guidance.nice.org.uk/IPG424)
2. Sooriakumaran P et al. Comparative effectiveness of radical prostatectomy and
radiotherapy in prostate cancer: observational study of mortality outcomes. BMJ
2014 (348):13. This study shows that in men with localised disease survival was
greater in those offered surgery.
Next question
Save my notes
Question stats
A 12.3%
B 14.4%
C 50.9%
D 13.2%
E 9.3%
Search eMRCS
Search term Go
Question 52 of 63
gathered by dr. elbarky, for free, not intended for profit by anybody elsewhere
A 23 year old woman is admitted with loin pain and a fever, she has noticed
haematuria for the past week accompanied by dysuria, this was treated empirically
with trimethoprim. What is the most likely cause?
Stone disease
Cystitis
Pyelonephritis
Renal cancer
Detrusor instability
This is most likely pyelonephritis and partially treated cystitis is a common cause.
Next question
Haematuria
Causes of haematuria
Infection Remember TB
Malignancy Renal cell carcinoma (remember paraneoplastic
syndromes): painful or painless
Urothelial malignancies: 90% are transitional cell
gathered by dr. elbarky, for free, not intended for profit by anybody elsewhere
Benign Exercise
Iatrogenic Catheterisation
Radiotherapy; cystitis, severe haemorrhage,
bladder necrosis
References
Http://bestpractice.bmj.com/best-practice/monograph/316/overview
/aetiology.html
Next question
gathered by dr. elbarky, for free, not intended for profit by anybody elsewhere
Save my notes
Question stats
A 14.4%
B 15.3%
C 59%
D 5.7%
E 5.6%
Search eMRCS
Search term Go
External links
+ Suggest a link
Dashboard
3
Question 53 of 63
gathered by dr. elbarky, for free, not intended for profit by anybody elsewhere
A 58 year old man has an episode of painless frank haematuria whilst undergoing
a 24 urine collection for investigation of hypertension. What is the most likely
cause?
Renal adenocarcinoma
Neuroblastoma
Phaeochromocytoma
Next question
Haematuria
Causes of haematuria
Infection Remember TB
Malignancy Renal cell carcinoma (remember paraneoplastic
syndromes): painful or painless
Urothelial malignancies: 90% are transitional cell
gathered by dr. elbarky, for free, not intended for profit by anybody elsewhere
Benign Exercise
Iatrogenic Catheterisation
Radiotherapy; cystitis, severe haemorrhage,
bladder necrosis
References
Http://bestpractice.bmj.com/best-practice/monograph/316/overview
/aetiology.html
Next question
gathered by dr. elbarky, for free, not intended for profit by anybody elsewhere
Save my notes
Question stats
A 47%
B 7.1%
C 16.1%
D 10.7%
E 19.1%
Search eMRCS
Search term Go
External links
+ Suggest a link
Dashboard
3
Question 54 of 63
gathered by dr. elbarky, for free, not intended for profit by anybody elsewhere
A 56 year old man is involved in a road traffic accident. He is found to have a pelvic
fracture. He reports that he has some lower abdominal pain. He has peritonism in
the lower abdomen. The nursing staff report that he has not passed any urine. A
CT scan shows evidence of free fluid. What is the most likely diagnosis?
Bladder rupture
Bladder contusion
Next question
Types of injury
Urethral injury Mainly in males
i.Bulbar rupture
- most common
- straddle type injury e.g. bicycles
- triad signs: urinary retention, perineal haematoma,
blood at the meatus
ii. Membranous rupture
- can be extra or intraperitoneal
- commonly due to pelvic fracture
- Penile or perineal oedema/ hematoma
- PR: prostate displaced upwards (beware co-existing
retroperitoneal haematomas as they may make
examination difficult)
Next question
Save my notes
gathered by dr. elbarky, for free, not intended for profit by anybody elsewhere
Question stats
A 19.5%
B 58.7%
C 6.9%
D 9.7%
E 5.3%
Search eMRCS
Search term Go
External links
+ Suggest a link
Dashboard
8
Question 55 of 63
gathered by dr. elbarky, for free, not intended for profit by anybody elsewhere
Insertion of nephrostomy
Lithotripsy
The likely scenario is that this man has developed a calculus causing ureteric
obstruction. The stagnant column of urine can become colonised and infected. An
infected obstructed system is one of the few true urological emergencies. A
nephrostomy is needed as the stone could not be removed.
Next question
Hydronephrosis
Causes of hydronephrosis
Unilateral: PACT
Pelvic-ureteric obstruction (congenital or acquired)
Aberrant renal vessels
Calculi
Tumours of renal pelvis
Bilateral: SUPER
Stenosis of the urethra
Urethral valve
gathered by dr. elbarky, for free, not intended for profit by anybody elsewhere
Prostatic enlargement
Extensive bladder tumour
Retro-peritoneal fibrosis
Investigation
USS- identifies presence of hydronephrosis and can assess the kidneys
IVU- assess the position of the obstruction
Antegrade or retrograde pyelography- allows treatment
If renal colic suspected: non contrast CT scan (majority of stones are
detected this way)
Management
Remove the obstruction and drainage of urine
Acute upper urinary tract obstruction: Nephrostomy tube
Chronic upper urinary tract obstruction: Ureteric stent or a pyeloplasty
Next question
Save my notes
Question stats
A 18.1%
B 9.5%
C 55.1%
D 8.3%
E 9%
Question 56 of 63
gathered by dr. elbarky, for free, not intended for profit by anybody elsewhere
A 75 year old lady reports urinary incontinence when coughing and sneezing. She
has had 2 children with no complications. She has no significant past medical
history and is on no medications. What is the most appropriate initial
management?
Urodynamic studies
Administration of oxybutinin
Next question
Urinary incontinence
Involuntary passage of urine. Most cases are female (80%). It has a prevalence of
11% in those aged greater than 65 years. The commonest variants include:
Stress urinary incontinence (50%)
Urge incontinence (15%)
Mixed (35%)
Males
Males may also suffer from incontinence although it is a much rarer condition in
men. A number of anatomical factors contribute to this. Males have 2 powerful
sphincters; one at the bladder neck and the other in the urethra. Damage to the
bladder neck mechanism is a factor in causing retrograde ejaculation following
prostatectomy. The short segment of urethra passing through the urogenital
diaphragm consists of striated muscle fibres (the external urethral sphincter) and
gathered by dr. elbarky, for free, not intended for profit by anybody elsewhere
Females
The sphincter complex at the level of bladder neck is poorly developed in females.
As a result the external sphincter complex is functionally more important, its
composition being similar to that of males. Innervation is via the pudendal nerve
and the neuropathy that may accompany obstetric events may compromise this
and lead to stress urinary incontinence.
Innervation
Somatic innervation to the bladder is via the pudendal, hypogastric and pelvic
nerves. Autonomic nerves travel in these nerve fibres too. Bladder filling leads to
detrusor relaxation (sympathetic) coupled with sphincter contraction. The
parasympathetic system causes detrusor contraction and sphincter relaxation.
Overall control of micturition is centrally mediated via centres in the Pons.
Urethral mobility:
Pressure not transmitted appropriately to the urethra resulting in involuntary
passage of urine during episodes of raised intra-abdominal pressure.
Sphincter dysfunction:
Sphincter fails to adapt to compress urethra resulting in involuntary passage of
urine. When the sphincter completely fails there is often to continuous passage of
urine.
Urge incontinence
In these patients there is sense of urgency followed by incontinence. The detrusor
muscle in these patients is unstable and urodynamic investigation will
demonstrate overactivity of the detrusor muscle at inappropriate times (e.g.
Bladder filling). Urgency may be seen in patients with overt neurological disorders
and those without. The pathophysiology is not well understood but poor central
and peripheral co-ordination of the events surrounding bladder filling are the main
processes.
Assessment
Careful history and examination including vaginal examination for cystocele.
Bladder diary for at least 3 days
Consider flow cystometry if unclear symptomatology or surgery considered and
diagnosis is unclear.
Exclusion of other organic disease (e.g. Stones, UTI, Cancer)
gathered by dr. elbarky, for free, not intended for profit by anybody elsewhere
Management
Conservative measures should be tried first; Stress urinary incontinence or mixed
symptoms should undergo 3 months of pelvic floor exercise. Over active bladder
should have 6 weeks of bladder retraining.
Drug therapy for women with overactive bladder should be offered oxybutynin (or
solifenacin if elderly) if conservative measures fail.
In women with detrusor instability who fail non operative therapy a trial of sacral
neuromodulation may be considered, with conversion to permanent implant if
good response. Augmentation cystoplasty is an alternative but will involve long
term intermittent self catheterisation.
In women with stress urinary incontinence a urethral sling type procedure may be
undertaken. Where cystocele is present in association with incontinence it should
be repaired particularly if it lies at the introitus.
NICE guidelines
Initial assessment urinary incontinence should be classified as stress/urge
/mixed.
At least 3/7 bladder diary if unable to classify easily.
Start conservative treatment before urodynamic studies if a diagnosis is
obvious from the history
Urodynamic studies if plans for surgery.
Stress incontinence: Pelvic floor exercises 3/12, if fails consider surgery.
Urge incontinence: Bladder training >6/52, if fails for oxybutynin
(antimuscarinic drugs) then sacral nerve stimulation.
Pelvic floor exercises offered to all women in their 1st pregnancy.
Next question
Save my notes
Question stats
A 6.6%
B 14.6%
C 63.5%
D 7.6%
gathered by dr. elbarky, for free, not intended for profit by anybody elsewhere
E 7.7%
Search eMRCS
Search term Go
External links
+ Suggest a link
Dashboard
10
11
12
13
Question 57 of 63
gathered by dr. elbarky, for free, not intended for profit by anybody elsewhere
Retroperitoneal sarcoma
Retroperitoneal fibrosis
Next question
Renal tumours
Investigation
Many cases will present as haematuria and be discovered during diagnostic work
gathered by dr. elbarky, for free, not intended for profit by anybody elsewhere
up. Benign renal tumours are rare, so renal masses should be investigated with
multislice CT scanning. Some units will add an arterial and venous phase to the
scan to demonstrate vascularity and evidence of caval ingrowth.
CT scanning of the chest and abdomen to detect distant disease should also be
undertaken.
Management
T1 lesions may be managed by partial nephrectomy and this gives equivalent
oncological results to total radical nephrectomy. Partial nephrectomy may also be
performed when there is inadequate reserve in the remaining kidney.
For T2 lesions and above a radical nephrectomy is standard practice and this may
be performed via a laparoscopic or open approach. Preoperative embolisation is
not indicated nor is resection of uninvolved adrenal glands. During surgery early
venous control is mandatory to avoid shedding of tumour cells into the circulation.
Patients with completely resected disease do not benefit from adjuvant therapy
with either chemotherapy or biological agents. These should not be administered
outside the setting of clinical trials.
References
Lungberg B et al. EAU guidelines on renal cell carcinoma: The 2010 update.
European Urology 2010 (58): 398-406.
Next question
Save my notes
gathered by dr. elbarky, for free, not intended for profit by anybody elsewhere
Question stats
A 14.5%
B 57%
C 12%
D 7.1%
E 9.5%
Search eMRCS
Search term Go
External links
+ Suggest a link
Dashboard
8
Question 58 of 63
gathered by dr. elbarky, for free, not intended for profit by anybody elsewhere
Bladder tuberculosis
Transitional cell carcinoma of the bladder may be treated with intravesical BCG
therapy.
Next question
Haematuria
Causes of haematuria
Infection Remember TB
Malignancy Renal cell carcinoma (remember paraneoplastic
syndromes): painful or painless
Benign Exercise
Iatrogenic Catheterisation
Radiotherapy; cystitis, severe haemorrhage,
bladder necrosis
References
Http://bestpractice.bmj.com/best-practice/monograph/316/overview
/aetiology.html
Next question
gathered by dr. elbarky, for free, not intended for profit by anybody elsewhere
Save my notes
Question stats
A 16.6%
B 55.7%
C 10%
D 11.6%
E 6.1%
Search eMRCS
Search term Go
External links
+ Suggest a link
Dashboard
1
Question 59 of 63
gathered by dr. elbarky, for free, not intended for profit by anybody elsewhere
A 52 year old man falls off his bike. He is found to have a pelvic fracture. On
examination, he is found to have perineal oedema and on PR the prostate is not
palpable. What is the most likely underlying problem?
Bladder rupture
Bladder contusion
Next question
Types of injury
Urethral injury Mainly in males
Blood at the meatus (50% cases)
i.Bulbar rupture
- most common
- straddle type injury e.g. bicycles
- triad signs: urinary retention, perineal haematoma,
blood at the meatus
ii. Membranous rupture
- can be extra or intraperitoneal
- commonly due to pelvic fracture
- Penile or perineal oedema/ hematoma
- PR: prostate displaced upwards (beware co-existing
retroperitoneal haematomas as they may make
examination difficult)
Next question
Save my notes
gathered by dr. elbarky, for free, not intended for profit by anybody elsewhere
Question stats
A 24.4%
B 6.7%
C 15.9%
D 7.2%
E 45.7%
Search eMRCS
Search term Go
External links
+ Suggest a link
Dashboard
8
Question 60 of 63
gathered by dr. elbarky, for free, not intended for profit by anybody elsewhere
A 38 year old man is investigated and found to have a left sided renal mass.
Imaging demonstrates a 5cm renal tumour invading the left renal vein. There is no
evidence of distant disease. What is the most appropriate course of action?
Renal biopsy
Radical nephrectomy
Combined chemoradiotherapy
Neoadjuvent chemotherapy
Renal tumours that are operable are nearly always managed surgically in the first
instance. It is not common practice to biopsy prior to surgery. They are not usually
radiosensitive.
Next question
Renal tumours
Investigation
Many cases will present as haematuria and be discovered during diagnostic work
up. Benign renal tumours are rare, so renal masses should be investigated with
multislice CT scanning. Some units will add an arterial and venous phase to the
scan to demonstrate vascularity and evidence of caval ingrowth.
CT scanning of the chest and abdomen to detect distant disease should also be
undertaken.
Management
T1 lesions may be managed by partial nephrectomy and this gives equivalent
oncological results to total radical nephrectomy. Partial nephrectomy may also be
performed when there is inadequate reserve in the remaining kidney.
For T2 lesions and above a radical nephrectomy is standard practice and this may
be performed via a laparoscopic or open approach. Preoperative embolisation is
not indicated nor is resection of uninvolved adrenal glands. During surgery early
venous control is mandatory to avoid shedding of tumour cells into the circulation.
Patients with completely resected disease do not benefit from adjuvant therapy
with either chemotherapy or biological agents. These should not be administered
outside the setting of clinical trials.
References
Lungberg B et al. EAU guidelines on renal cell carcinoma: The 2010 update.
European Urology 2010 (58): 398-406.
Next question
Save my notes
Question stats
A 19.1%
B 55.5%
C 7.1%
D 9.4%
E 9%
Search eMRCS
Search term Go
External links
+ Suggest a link
Dashboard
7
Question 61 of 63
gathered by dr. elbarky, for free, not intended for profit by anybody elsewhere
Chemotherapy alone
Radical prostatectomy
Trasvesical prostatectomy
TURP
In a young patient with local disease only a radical prostatectomy is the best
chance of cure. Radiotherapy may be given instead but has long term sequelae
(and inferior survival outcomes). A transvesical prostatectomy is a largely
historical operation performed for BPH before TURP was established.
Next question
Prostate Cancer
Prostate Cancer
This is a common condition and up to 30,000 men are diagnosed with the
condition each year. Up to 9,000 will die in in the UK from the condition per year.
Diagnosis
Early prostate cancers have few symptoms.
Metastatic disease may present as bone pain.
Locally advanced disease may present as pelvic pain or with urinary symptoms.
Prostate specific antigen measurement
Digital rectal examination
Trans rectal USS (+/- biopsy)
MRI/ CT and bone scan for staging.
PSA Test
gathered by dr. elbarky, for free, not intended for profit by anybody elsewhere
The normal upper limit for PSA is 4ng/ml. However, in this group will lie patients
with benign disease and some with localised prostate cancer. False positives may
be due to prostatitis, UTI, BPH, vigorous DRE.
The percentage of free: total PSA may help to distinguish benign disease from
cancer. Values of <20% are suggestive of cancer and biopsy is advised.
Pathology
95% adenocarcinoma
In situ malignancy is sometimes found in areas adjacent to cancer. Multiple
biopsies needed to call true in situ disease.
Often multifocal- 70% lie in the peripheral zone.
Graded using the Gleason grading system, two grades awarded 1 for most
dominant grade (on scale of 1-5) and 2 for second most dominant grade
(scale 1-5). The two added together give the Gleason score. Where 2 is best
prognosis and 10 the worst.
Lymphatic spread occurs first to the obturator nodes and local extra
prostatic spread to the seminal vesicles is associated with distant disease.
Treatment
Watch and wait- Elderly, multiple co-morbidities, low Gleason score
Radiotherapy (External)- Both potentially curative and palliative therapy
possible. However, radiation proctitis and rectal malignancy are late
problems. Brachytherapy is a modification allowing internal radiotherapy.
Surgery- Radical prostatectomy. Surgical removal of the prostate is the
standard treatment for localised disease. The robot is being used
increasingly for this procedure. As well as the prostate the obturator nodes
are also removed to complement the staging process. Erectile dysfunction
is a common side effect. Survival may be better than with radiotherapy (see
references).
Hormonal therapy- Testosterone stimulates prostate tissue and prostatic
cancers usually show some degree of testosterone dependence. 95% of
testosterone is derived from the testis and bilateral orchidectomy may be
used for this reason. Pharmacological alternatives include LHRH analogues
and anti androgens (which may be given in combination).
In the UK the National Institute for Clinical Excellence (NICE) suggests that
active surveillance is the preferred option for low risk men. It is particularly
suitable for men with clinical stage T1c, Gleason score 3+3 and PSA density
< 0.15 ng/ml/ml who have cancer in less than 50% of their biopsy cores,
with < 10 mm of any core involved.
References
gathered by dr. elbarky, for free, not intended for profit by anybody elsewhere
Next question
Save my notes
Question stats
A 6.8%
B 49.4%
C 11.8%
D 21.1%
E 10.9%
Search eMRCS
Search term Go
External links
Question 62 of 63
gathered by dr. elbarky, for free, not intended for profit by anybody elsewhere
A 32 year old lady suffers from severe left sided abdominal pain, that radiates to
her groin. As part of her evaluation the nurses identify microscopic haematuria on
dipstick. What is the explanation?
Staghorn calculus
Ureteric calculus
Cystitis
Ureteric calculi will often present with loin pain radiating to the groin. It is usually
severe. There may be macroscopic or microscopic haematuria. The absence of
haematuria on dipstick testing should prompt investigations for alternative
diagnoses. The best investigation is a non contrast CT scan. CT changes
consistent with stone or recent stone passage include evidence of stone,
perinephric stranding, ureteric oedema or hydronephrosis.
Next question
Haematuria
Causes of haematuria
Infection Remember TB
References
Http://bestpractice.bmj.com/best-practice/monograph/316/overview
Malignancy Renal cell carcinoma (remember paraneoplastic
/aetiology.html syndromes): painful or painless
Urothelial malignancies: 90% are transitional cell
carcinoma, can occur anywhere along the urinary
tract. Painless haematuria. Next question
Squamous cell carcinoma and adenocarcinoma:
rare bladder tumours
Prostate cancer
Penile cancers: SCC
Display my notes on this topic
Renal
disease
Glomerulonephritis
Save my notes
Structural Benign prostatic hyperplasia (BPH) causes
abnormalities haematuria
Question due
statsto hypervascularity of the
prostate gland
Cystic renal lesions e.g. polycystic kidney disease
A 12.5%
Vascular malformations
B 64.2%
Renal vein thrombosis due to renal cell carcinoma
C 8.3%
Coagulopathy
D Causes bleeding of underlying lesions
6%
E 9.1%
Drugs Cause tubular necrosis or interstitial nephritis:
aminoglycosides, chemotherapy
64.2% of users answered this question correctly
Interstitial nephritis: penicillin, sulphonamides,
and NSAIDs
Anticoagulants
Search eMRCS
Benign Exercise
Search term Go
Gynaecological Endometriosis: flank pain, dysuria, and
haematuria that is cyclical
Iatrogenic Catheterisation
Question 63 of 63
gathered by dr. elbarky, for free, not intended for profit by anybody elsewhere
A 78 year old man has a long history of nocturia, urinary frequency and terminal
dribbling. He was admitted with urinary retention and was catheterised. On
removal of the catheter he has noticed some haematuria. What is the most likely
cause?
BPH is increasingly common with advancing age and is present in 50% over the
age of 60 and nearly 90% of men by the age of 90. It occurs as a result of
hyperplasia of the periurethral glands in the transitional zone of the prostate.
Androgens play a role in the development and progression of BPH. Testosterone
diffuses into prostatic and stromal cells. Within epithelial cells it binds to the
androgen receptor. In prostatic stromal cells, a small proportion binds directly to
the androgen receptor, the majority binds to the 5 alpha reductase type II receptor
on the nuclear membrane. This is converted to dihyroxytestosterone and then
binds to the androgen receptor. Dihydroxytestosterone has even greater affinity for
the androgen receptor than testosterone does. The end result is stimulation of
these cells and proliferation.
This proliferative activity results in varying degrees of obstruction and results in
lower urinary tract obstructive symptoms. The clinical diagnosis of BPH thus
comprise a degree of lower urinary tract symptoms, palpable prostatic
enlargement and evidence of impaired voiding on urodynamic assessment.
Presentation
gathered by dr. elbarky, for free, not intended for profit by anybody elsewhere
Diagnosis
Abdominal and rectal examination
Symptoms scoring
Urodynamic studies ( a post void volume of >100ml is significant)
Management
Conservative
Alpha adrenergic antagonists. These block the action of noradrenaline on prostatic
smooth muscle causing relaxation and improved bladder emptying.
5 alpha reductase inhibitors. Finasteride blocks the enzyme 5 alpha reductase
which inhibits the conversion of testosterone to DHT. This in turn reduces
intracellular activity and decreases prostatic volume
Surgery- TURP is the gold standard. Occasionally, an open retropubic
prostatectomy may be considered for a large gland.
Save my notes
Question stats
A 14%
B 9.1%
C 6.9%
D 6.5%
E 63.4%